Exam One Flashcards

1
Q
A nurse is caring for a young adult. Which goal is priority?
a.
Maintain peer relationships.
b.
Maintain family relationships.
c.
Maintain parenteral relationships.
d.
Maintain recreational relationships.
A

ANS: B
Family is important during young adulthood. Challenges may include the demands of working and raising families. Peer is more important in the adolescent years. Young adults are much freer from parental control. While recreation is important, the family and work are the priorities in young adults.

How well did you know this?
1
Not at all
2
3
4
5
Perfectly
2
Q

The nurse is caring for a hospitalized young-adult male who works as a dishwasher at a local restaurant. He states that he would like to get a better job but has no education. How can the nurse best assist this patient psychosocially?
a.
By providing information and referrals
b.
By focusing on the patient’s medical diagnoses
c.
By telling the patient that he needs to go back to school
d.
By expecting the patient to be flexible in decision making

A

ANS: A
Support from the nurse, access to information, and appropriate referrals provide opportunities for achievement of a patient’s potential. Health is not merely the absence of disease (focusing on medical diagnoses) but involves wellness in all human dimensions. Telling a patient what to do (go back to school) is inappropriate. Each person (not the nurse) needs to make these decisions based on individual factors. Insecure persons tend to be more rigid in making decisions.

How well did you know this?
1
Not at all
2
3
4
5
Perfectly
3
Q
Which goal is priority when the nurse is caring for a middle-aged adult?
a.
Maintain immediate family relationships.
b.
Maintain future generation relationships.
c.
Maintain personal career relationships.
d.
Maintain work relationships.
A

ANS: B
Many middle-aged adults find particular joy in helping their children and other young people become productive and responsible adults. While immediate family is important, this goal is priority in young adults, not as important in middle-aged adults. During this period, personal and career achievements have often already been experienced; therefore, these goals are not priority.

How well did you know this?
1
Not at all
2
3
4
5
Perfectly
4
Q

A nurse is teaching young adults about health risks. Which statement from a young adult indicates a correct understanding of the teaching?
a.
“It’s probably safe for me to start smoking. At my age, there’s not enough time for cancer to develop.”
b.
“My mother had appendicitis, so this increases my chance for developing appendicitis.”
c.
“Controlling the amount of stress in my life may decrease the risk of illness.”
d.
“I don’t do drugs. I do drink coffee, but caffeine is not a drug.”

A

ANS: C
Lifestyle habits that activate the stress response increase the risk of illness; so, controlling this will decrease risk. Smoking is a well-documented risk factor for pulmonary, cardiac, and vascular disease as well as cancer in smokers and in individuals who receive secondhand smoke. The presence of certain chronic illnesses (not acute illnesses—appendicitis) in the family increases the family member’s risk of developing a disease. Caffeine is a naturally occurring legal stimulant that is readily available. Caffeine stimulates catecholamine release, which, in turn, stimulates the central nervous system; it also increases gastric acid secretion, heart rate, and basal metabolic rate.

How well did you know this?
1
Not at all
2
3
4
5
Perfectly
5
Q
A nurse is choosing an appropriate topic for a young-adult health fair. Which topic should the nurse include?
a.
Retirement
b.
Menopause
c.
Climacteric factors
d.
Unplanned pregnancies
A

ANS: D
Unplanned pregnancies are a continued source of stress that can result in adverse health outcomes for the mother (young adult), infant, and family. Retirement is an issue for middle-aged, not young adults. The onset of menopause and the climacteric affect the sexual health of the middle-aged adult, not the young adult.

How well did you know this?
1
Not at all
2
3
4
5
Perfectly
6
Q
A nurse is assessing the risk of intimate partner violence (IPV) for patients. Which population should the nurse focus on most for IVP?
a.
White males
b.
Pregnant females
c.
Middle-aged adults
d.
Nonsubstance abusers
A

ANS: B
The greatest risk of violence occurs during the reproductive years. A pregnant woman has a 35.6% greater risk of being a victim of IPV than a nonpregnant woman. White males, middle-aged adults, and nonsubstance abusers are not as high risk as pregnant women.

How well did you know this?
1
Not at all
2
3
4
5
Perfectly
7
Q

A patient states that she is pregnant and concerned because she does not know what to expect, and she wants her partner to play an active part in the birthing process. Which information should the nurse share with the patient?
a.
Lamaze classes can prepare pregnant women and their partners for what is coming.
b.
The frequency of sexual intercourse is key to helping the husband feel valued.
c.
After the birth, the stress of pregnancy will disappear and will be replaced by relief.
d.
After the baby is born, the wife should accept the extra responsibilities of motherhood.

A

ANS: A
Childbirth education classes (like Lamaze) can prepare pregnant women, their partners, and other support persons to participate in the birthing process. The psychodynamic aspect of sexual activity is as important as the type or frequency of sexual intercourse to young adults; however, this does not relate to the issue the patient reports (lack of knowledge and participation). The stress that many women experience after childbirth has a significant impact on the health of postpartum women. Ideally partners should share all responsibilities; however, this does not relate to the patient’s concerns.

How well did you know this?
1
Not at all
2
3
4
5
Perfectly
8
Q
Which information from the nurse indicates a correct understanding of emerging adulthood?
a.
It is a type of young adulthood.
b.
It is a type of extended adolescence.
c.
It is a type of independent exploration.
d.
It is a type of marriage and parenthood.
A

ANS: C
This newly identified stage of development from age 18 to 25 (emerging adulthood) has been described as neither an extended adolescence, as it is much freer from parental control and is much more a period of independent exploration, nor young adulthood, as most young people in their twenties have not made the transitions historically associated with adult status, especially marriage and parenthood.

How well did you know this?
1
Not at all
2
3
4
5
Perfectly
9
Q
A nurse is planning care for a 30 year old. Which goal is priority?
a.
Refine self-perception.
b.
Master career plans.
c.
Examine life goals.
d.
Achieve intimacy.
A

ANS: B
From 29 to 34, the person directs enormous energy toward achievement and mastery of the surrounding world. The years from 35 to 43 are a time of vigorous examination of life goals and relationships. Between the ages of 23 and 28, the person refines self-perception and ability for intimacy.

How well did you know this?
1
Not at all
2
3
4
5
Perfectly
10
Q

A nurse is planning care for young-adult patients. Which information should the nurse consider when planning care?
a.
Fertility issues do not occur in young adulthood.
b.
Young adults tend to suffer more from severe illness.
c.
Substance abuse is easy to observe in young-adult patients.
d.
Young adults are quite active but are at risk for illness in later years

A

ANS: D
Young adults are generally active and experience severe illnesses less frequently. However, their lifestyles may put them at risk for illnesses or disabilities during their middle or older-adult years. An estimated 10% to 15% of reproductive couples are infertile, and many are young adults. Substance abuse is not always diagnosable, particularly in its early stages.

How well did you know this?
1
Not at all
2
3
4
5
Perfectly
11
Q

During a routine physical assessment, the nurse obtaining a health history notes that a 50-year-old female patient reports pain and redness in the right breast. Which action is best for the nurse to take in response to this finding?
a.
Assess the patient as thoroughly as possible.
b.
Explain to the patient that breast tenderness is normal at her age.
c.
Tell the patient that redness is not a cause for concern and is quite common.
d.
Inform her that redness is the precursor to normal unilateral breast enlargement.

A

ANS: A
A comprehensive assessment offers direction for health promotion recommendations, as well as for planning and implementing any acutely needed intervention. Redness or painful breasts are abnormal physical assessment findings in the middle-aged adult. Increased size of one breast is an abnormal physical assessment finding in the middle-aged adult.

How well did you know this?
1
Not at all
2
3
4
5
Perfectly
12
Q

A 55-year-old female presents to the outpatient clinic describing irregular menstrual periods and hot flashes. Which information should the nurse share with the patient?
a.
The patient’s assessment points toward normal menopause.
b.
Those symptoms are normal when a woman undergoes the climacteric.
c.
An assessment is not really needed because these problems are normal for older women.
d.
The patient should stop regular exercise because that is probably causing these symptoms.

A

ANS: A
The most significant physiological changes during middle age are menopause in women and the climacteric in men. Menopause typically occurs between 45 and 60 years of age. The nurse should continue with the examination because a comprehensive assessment offers direction for health promotion recommendations, as well as for planning and implementing any acutely needed interventions. Exercise should not be stopped, especially in middle-aged adults.

How well did you know this?
1
Not at all
2
3
4
5
Perfectly
13
Q

The nurse is teaching a class to pregnant women about common physiological changes during pregnancy. Which information should the nurse include in the teaching session?
a.
Pregnancy is not a time to be having sexual activity.
b.
Urinary frequency will occur early in the pregnancy.
c.
Breast tenderness should be reported as soon as possible.
d.
Late in the pregnancy Braxton Hicks contraction may occur.

A

ANS: D
During the third trimester (late pregnancy), increases in Braxton Hicks contractions (irregular, short contractions), fatigue, and urinary frequency (not early) occur. Normally, women commonly have morning sickness, breast enlargement and tenderness, and fatigue. Women need to be reassured that sexual activity will not harm the fetus.

How well did you know this?
1
Not at all
2
3
4
5
Perfectly
14
Q

A nurse discusses the risks of repeated sun exposure with a young-adult patient. Which response will the nurse most expect from this patient?
a.
“I should consider participating in a health fair about safe sun practices.”
b.
“I’ll make an appointment with my doctor right away for a full skin check.”
c.
“I’ve had this mole my whole life. So, what if it changed color? My skin is fine.”
d.
“I have a mole that has been bothering me. I’ll call my family doctor for an appointment to get it checked.”

A

ANS: C
Most typically young adults would say that their skin is fine. Young adults often ignore physical symptoms and often postpone seeking health care. Making an appointment right away with the doctor and participating in health fairs are not typical behaviors of young adults for the same reason.

How well did you know this?
1
Not at all
2
3
4
5
Perfectly
15
Q
Upon assessment of a middle-aged adult, the nurse observes uneven weight bearing and decreased range of joint motion. Which area is priority?
a.
Abuse potential
b.
Fall precautions
c.
Stroke prevention
d.
Self-esteem issues
A

ANS: B
With uneven weight bearing and decreased range of joint motion, falling is a priority. Abuse potential would indicate other findings such as bruising or unkept appearance. While stroke prevention is important in a middle-aged adult, these are not the signs of stroke. While self-esteem issues may arise from physical changes, safety is a priority over self-esteem issues.

How well did you know this?
1
Not at all
2
3
4
5
Perfectly
16
Q

A young-adult patient is brought to the hospital by police after crashing the car in a high-speed chase when trying to avoid arrest for spousal abuse. Which action should the nurse take?
a.
Question the patient about drug use.
b.
Offer the patient a cup of coffee to calm nerves.
c.
Discretely assess the patient for sexually transmitted infections.
d.
Deal with the issue at hand, not asking about previous illnesses.

A

ANS: A
Reports of arrests because of driving while intoxicated, wife or child abuse, or disorderly conduct are reasons for the nurse to investigate the possibility of drug abuse more carefully. Caffeine is a naturally occurring legal stimulant that stimulates the central nervous system and is not the choice for calming nerves. Although sexually transmitted infections occur in the young adult, this is not an action a nurse should take in this situation. The nurse may obtain important information by making specific inquiries about past medical problems, changes in food intake or sleep patterns, and problems of emotional lability.

How well did you know this?
1
Not at all
2
3
4
5
Perfectly
17
Q

A nurse determines that a middle-aged patient is a typical example of the “sandwich generation.” What did the nurse discover the patient is caught between?
a.
Job responsibilities or family responsibilities
b.
Stopping old habits and starting new ones
c.
Caring for children and aging parents
d.
Advancing in career or retiring

A

ANS: C
Middle-aged adults also begin to help aging parents while being responsible for their own children, placing them in the sandwich generation. It does not include job and family responsibilities; old habits and new ones; or career and retiring.

How well did you know this?
1
Not at all
2
3
4
5
Perfectly
18
Q
A nurse is assessing a middle-aged patient’s barriers to change in eating habits. Which areas will the nurse assess that are external barriers? (Select all that apply.)
a.
Lack of facilities
b.
Lack of materials
c.
Lack of knowledge
d.
Lack of social supports
e.
Lack of short- and long-term goals
A

ANS: A, B, D
External barriers to change include lack of facilities, materials, and social supports. Internal barriers are lack of knowledge, insufficient skills, and undefined short- and long-term goals.

How well did you know this?
1
Not at all
2
3
4
5
Perfectly
19
Q
A home health nurse is providing care to a middle-aged couple with children at home. The patient has a debilitating chronic illness. Which areas will the nurse need to assess? (Select all that apply.)
a.
Adherence to treatment and rehabilitation regimens
b.
Coping mechanisms of patient and family
c.
Need for community services or referrals
d.
Knowledge base of patient only
e.
Use of a doula for care
A

ANS: A, B, C
Along with the current health status of the chronically ill middle-aged adult, you need to assess the knowledge base of both the patient and family. In addition, you must determine the coping mechanisms of the patient and family, adherence to treatment and rehabilitation regimens, and the need for community and social services, along with appropriate referrals. A doula is a support person to be present during labor to assist women who have no other source of support.

How well did you know this?
1
Not at all
2
3
4
5
Perfectly
20
Q
A nurse is providing prenatal care to a first-time mother. Which information will the nurse share with the patient? (Select all that apply.)
a.
Regular trend for postpartum depression
b.
Protection against urinary infection
c.
Strategies for empty nest syndrome
d.
Exercise patterns
e.
Proper diet
A

ANS: B, D, E
Prenatal care includes a thorough physical assessment of the pregnant woman during regularly scheduled intervals. Information regarding STIs and other vaginal infections and urinary infections that will adversely affect the fetus and counseling about exercise patterns, diet, and child care are important for a pregnant woman. Empty nest syndrome occurs as children leave the home. Postpartum depression is rare.

How well did you know this?
1
Not at all
2
3
4
5
Perfectly
21
Q
A nurse is obtaining a history on an older adult. Which finding will the nurse most typically find?
a.
Lives in a nursing home.
b.
Lives with a spouse.
c.
Lives divorced.
d.
Lives alone.
A

ANS: B
Recent research found that 59% of older adults in noninstitutional settings lived with a spouse (48% of older women, 72% of older men); 28% lived alone (34% of older women, 20% of older men); and only 3.1% of all older adults resided in institutions such as nursing homes or centers. Most older adults have lost a spouse due to death rather than divorce.

How well did you know this?
1
Not at all
2
3
4
5
Perfectly
22
Q

A nurse is developing a plan of care for an older adult. Which information will the nurse consider?
a.
Should be standardized because most geriatric patients have the same needs.
b.
Needs to be individualized to the patient’s unique needs.
c.
Focuses on the disabilities that all aging persons face.
d.
Must be based on chronological age alone.

A

ANS: B
Every older adult is unique, and the nurse needs to approach each one as a unique individual. The nursing care of older adults poses special challenges because of great variation in their physiological, cognitive, and psychosocial health. Aging does not automatically lead to disability and dependence. Chronological age often has little relation to the reality of aging for an older adult

How well did you know this?
1
Not at all
2
3
4
5
Perfectly
23
Q

Which information presented by a co-worker on a gerontological unit will cause the nurse to intervene?
a.
Most older people have dependent functioning.
b.
Most older people have strengths we should focus on.
c.
Most older people should be involved in care decision.
d.
Most older people should be encouraged to have independence.

A

ANS: A
Most older people remain functionally independent despite the increasing prevalence of chronic disease; therefore, this misconception should be addressed. It is critical for you to respect older adults and actively involve them in care decisions and activities. You also need to identify an older adult’s strengths and abilities during the assessment and encourage independence as an integral part of your plan of care.

How well did you know this?
1
Not at all
2
3
4
5
Perfectly
24
Q
A nurse suspects an older-adult patient is experiencing caregiver neglect. Which assessment findings are consistent with the nurse’s suspicions?
a.
Flea bites and lice infestation.
b.
Left at a grocery store.
c.
Refuses to take a bath.
d.
Cuts and bruises.
A

ANS: A
Neglect is the failure to provide basic necessities such as food, water, shelter, hygiene, and medical care. Caregiver neglect includes unsafe and unclean living conditions, soiled bedding, and animal or insect infestation. Abandonment includes desertion at a hospital, nursing facility, or public location such as a shopping center. Self-neglect includes refusal or failure to provide oneself with basic necessities such as food, water, clothing, shelter, personal hygiene, medication, and safety. Physical abuse includes hitting, beating, pushing, slapping, kicking, physical restraint, inappropriate use of drugs, fractures, lacerations, rope burns, and untreated injuries.

How well did you know this?
1
Not at all
2
3
4
5
Perfectly
25
Q

A nurse is teaching a group of older-adult patients. Which teaching strategy is best for the nurse to use?
a.
Provide several topics of discussion at once to promote independence and making choices.
b.
Avoid uncomfortable silences after questions by helping patients complete their statements.
c.
Ask patients to recall past experiences that correspond with their interests.
d.
Speak in a high pitch to help patients hear better.

A

ANS: C
Teaching strategies include the use of past experiences to connect new learning with previous knowledge, focusing on a single topic to help the patient concentrate, giving the patient enough time in which to respond because older adults’ reaction times are longer than those of younger persons, and keeping the tone of voice low; older adults are able to hear low sounds better than high-frequency sounds.

How well did you know this?
1
Not at all
2
3
4
5
Perfectly
26
Q

An older patient has fallen and suffered a hip fracture. As a consequence, the patient’s family is concerned about the patient’s ability to care for self, especially during this convalescence. What should the nurse do?
a.
Stress that older patients usually ask for help when needed.
b.
Inform the family that placement in a nursing center is a permanent solution.
c.
Tell the family to enroll the patient in a ceramics class to maintain quality of life.
d.
Provide information and answer questions as family members make choices among care options

A

ANS: D
Nurses help older adults and their families by providing information and answering questions as they make choices among care options. Some older adults deny functional declines and refuse to ask for assistance with tasks that place their safety at great risk. The decision to enter a nursing center is never final, and a nursing center resident sometimes is discharged to home or to another less-acute residence. What defines quality of life varies and is unique for each person.

How well did you know this?
1
Not at all
2
3
4
5
Perfectly
27
Q

What is the best suggestion a nurse could make to a family requesting help in selecting a local nursing center?
a.
Have the family members evaluate nursing home staff according to their ability to get tasks done efficiently and safely.
b.
Make sure that nursing home staff members get patients out of bed and dressed according to staff’s preferences.
c.
Explain that it is important for the family to visit the center and inspect it personally.
d.
Suggest a nursing center that has standards as close to hospital standards as possible.

A

ANS: C
An important step in the process of selecting a nursing home is to visit the nursing home. The nursing home should not feel like a hospital. It is a home, a place where people live. Members of the nursing home staff should focus on the person, not the task. Residents should be out of bed and dressed according to their preferences, not staff preferences.

How well did you know this?
1
Not at all
2
3
4
5
Perfectly
28
Q

A 70-year-old patient who is experiencing worsening dementia is no longer able to live alone. The nurse is discussing health care services and possible long-term living arrangements with the patient’s only son. What will the nurse suggest?
a.
An apartment setting with neighbors close by.
b.
Having the patient utilize weekly home health visits.
c.
A nursing center because home care is no longer safe.
d.
That placement is irrelevant because the patient is retreating to a place of inactivity.

A

ANS: C
Some family caregivers consider nursing center placement when in-home care becomes increasingly difficult or when convalescence from hospitalization requires more assistance than the family is able to provide. An apartment setting and the use of home health visits are not appropriate because living at home is unsafe. Dementia is not a time of inactivity but an impairment of intellectual functioning.

How well did you know this?
1
Not at all
2
3
4
5
Perfectly
29
Q
A nurse is caring for an older adult. Which goal is priority?
a.
Adjusting to career
b.
Adjusting to divorce
c.
Adjusting to retirement
d.
Adjusting to grandchildren
A

ANS: C
Adjusting to retirement is one of the developmental tasks for an older person. A young or middle-aged adult has to adjust to career and/or divorce. A middle-aged adult has to adjust to grandchildren.

How well did you know this?
1
Not at all
2
3
4
5
Perfectly
30
Q
A nurse is observing for the common loss in an older-adult patient. What is the nurse assessing?
a.
Loss of finances through changes in income
b.
Loss of relationships through death
c.
Loss of career through retirement
d.
Loss of home through relocation
A

ANS: B
The universal loss for older adults usually revolves around the loss of relationships through death. Life transitions, of which loss is a major component, include retirement and the associated financial changes, changes in roles and relationships, alterations in health and functional ability, changes in one’s social network, and relocation. However, these are not the universal loss.

How well did you know this?
1
Not at all
2
3
4
5
Perfectly
31
Q

A nurse is discussing sexuality with an older adult. Which action will the nurse take?
a.
Ask closed-ended questions about specific symptoms the patient may experience.
b.
Provide information about the prevention of sexually transmitted infections.
c.
Discuss the issues of sexuality in a group in a private room.
d.
Explain that sexuality is not necessary as one ages.

A

ANS: B
Include information about the prevention of sexually transmitted infections when appropriate. Open-ended questions inviting an older adult to explain sexual activities or concerns elicit more information than a list of closed-ended questions about specific activities or symptoms. You need to provide privacy for any discussion of sexuality and maintain a nonjudgmental attitude. Sexuality and the need to express sexual feelings remain throughout the human life span.

How well did you know this?
1
Not at all
2
3
4
5
Perfectly
32
Q
A nurse is observing skin integrity of an older adult. Which finding will the nurse document as a normal finding?
a.
Oily skin
b.
Faster nail growth
c.
Decreased elasticity
d.
Increased facial hair in men
A

ANS: C
Loss of skin elasticity is a common finding in the older adult. Other common findings include pigmentation changes, glandular atrophy (oil, moisture, and sweat glands), thinning hair (facial hair: decreased in men, increased in women), slower nail growth, and atrophy of epidermal arterioles.

How well did you know this?
1
Not at all
2
3
4
5
Perfectly
33
Q

An older-adult patient in no acute distress reports being less able to taste and smell. What is the nurse’s best response to this information?
a.
Notify the health care provider immediately to rule out cranial nerve damage.
b.
Schedule the patient for an appointment at a smell and taste disorders clinic.
c.
Perform testing on the vestibulocochlear nerve and a hearing test.
d.
Explain to the patient that diminished senses are normal findings.

A

ANS: D
Diminished taste and smell senses are common findings in older adults. Scheduling an appointment at a smell and taste disorders clinic, testing the vestibulocochlear nerve, or an attempt to rule out cranial nerve damage is unnecessary at this time as per the information provided.

How well did you know this?
1
Not at all
2
3
4
5
Perfectly
34
Q
A nurse is assessing an older adult for cognitive changes. Which symptom will the nurse report as normal?
a.
Disorientation
b.
Poor judgment
c.
Slower reaction time
d.
Loss of language skills
A

ANS: C
Slower reaction time is a common change in the older adult. Symptoms of cognitive impairment, such as disorientation, loss of language skills, loss of the ability to calculate, and poor judgment are not normal aging changes and require further investigation of underlying causes.

How well did you know this?
1
Not at all
2
3
4
5
Perfectly
35
Q
An older patient diagnosed with dementia and confusion is admitted to the nursing unit after hip replacement surgery. Which action will the nurse include in the plan of care?
a.
Maintain a routine.
b.
Continue to reorient.
c.
Allow several choices.
d.
Socially isolate patient.
A

ANS: A
Patients experiencing dementia need a routine. Continuing to reorient a patient with dementia is nonproductive and not advised. Patients with dementia need limited choices. Social interaction based on the patient’s abilities is to be promoted.

How well did you know this?
1
Not at all
2
3
4
5
Perfectly
36
Q
A nurse is helping an older-adult patient with instrumental activities of daily living. The nurse will most likely be assisting the patient with which activity?
a.
Taking a bath
b.
Getting dressed
c.
Making a phone call
d.
Going to the bathroom
A

ANS: C
Instrumental activities of daily living or IADLs (such as the ability to write a check, shop, prepare meals, or make phone calls) and activities of daily living or ADLs (such as bathing, dressing, and toileting) are essential to independent living.

How well did you know this?
1
Not at all
2
3
4
5
Perfectly
37
Q

A male older-adult patient expresses concern and anxiety about decreased penile firmness during an erection. What is the nurse’s best response?
a.
Tell the patient that libido will always decrease, as well as the sexual desires.
b.
Tell the patient that touching should be avoided unless intercourse is planned.
c.
Tell the patient that heterosexuality will help maintain stronger libido.
d.
Tell the patient that this change is expected in aging adults

A

ANS: D
Aging men typically experience an erection that is less firm and shorter acting and have a less forceful ejaculation. Testosterone lessens with age and sometimes (not always) leads to a loss of libido. However, for both men and women sexual desires, thoughts, and actions continue throughout all decades of life. Sexuality involves love, warmth, sharing, and touching, not just the act of intercourse. Touch complements traditional sexual methods or serves as an alternative sexual expression when physical intercourse is not desired or possible. Clearly not all older adults are heterosexual, and there is emerging research on older adult, lesbian, gay, bisexual, and transgender individuals and their health care needs.

How well did you know this?
1
Not at all
2
3
4
5
Perfectly
38
Q

A patient asks the nurse what the term polypharmacy means. Which information should the nurse share with the patient?
a.
This is multiple side effects experienced when taking medications.
b.
This is many adverse drug effects reported to the pharmacy.
c.
This is the multiple risks of medication effects due to aging.
d.
This is concurrent use of many medications.

A

ANS: D
Polypharmacy refers to the concurrent use of many medications. It does not have anything to do with side effects, adverse drug effects, or risks of medication use due to aging.

How well did you know this?
1
Not at all
2
3
4
5
Perfectly
39
Q

An outcome for an older-adult patient living alone is to be free from falls. Which statement indicates the patient correctly understands the teaching on safety concerns?
a.
“I’ll take my time getting up from the bed or chair.”
b.
“I should dim the lighting outside to decrease the glare in my eyes.”
c.
“I’ll leave my throw rugs in place so that my feet won’t touch the cold tile.”
d.
“I should wear my favorite smooth bottom socks to protect my feet when walking around.”

A

ANS: A
Postural hypotension is an intrinsic factor that can cause falls. Changing positions slowly indicates a correct understanding of this concept. Environmental hazards outside and within the home such as poor lighting, slippery or wet flooring, and items on floor that are easy to trip over such as throw rugs are other factors that can lead to falls. Impaired vision and poor lighting are other risk factors for falls and should be avoided (dim lighting). Inappropriate footwear such as smooth bottom socks also contributes to falls.

How well did you know this?
1
Not at all
2
3
4
5
Perfectly
40
Q

A nurse’s goal for an older adult is to reduce the risk of adverse medication effects. Which action will the nurse take?
a.
Review the patient’s list of medications at each visit.
b.
Teach that polypharmacy is to be avoided at all cost.
c.
Avoid information about adverse effects.
d.
Focus only on prescribed medications.

A

ANS: A
Strategies for reducing the risk for adverse medication effects include reviewing the medications with older adults at each visit, examining for potential interactions with food or other medications, simplifying and individualizing medication regimens, taking every opportunity to inform older adults and their families about all aspects of medication use, and encouraging older adults to question their health care providers about all prescribed and over-the-counter medications. Although polypharmacy often reflects inappropriate prescribing, the concurrent use of multiple medications is often necessary when an older adult has multiple acute and chronic conditions. Older adults are at risk for adverse drug effects because of age-related changes in the absorption, distribution, metabolism, and excretion of drugs. Work collaboratively with the older adult to ensure safe and appropriate use of all medications—both prescribed medications and over-the-counter medications and herbal options.

How well did you know this?
1
Not at all
2
3
4
5
Perfectly
41
Q

An older-adult patient has developed acute confusion. The patient has been taking tranquilizers for the past week. The patient’s vital signs are normal. What should the nurse do?
a.
Consider age-related changes in body systems that affect pharmacokinetic activity.
b.
Increase the dose of tranquilizer if the cause of the confusion is an infection.
c.
Note when the confusion occurs and medicate before that time.
d.
Restrict phone calls to prevent further confusion.

A

ANS: A
Some sedatives and tranquilizers prescribed for acutely confused older adults sometimes cause or exacerbate confusion. Carefully administer drugs used to manage confused behaviors, considering age-related changes in body systems that affect pharmacokinetic activity. When confusion has a physiological cause (such as an infection), specifically treat that cause, rather than the confused behavior. When confusion varies by time of day or is related to environmental factors, nonpharmacological measures such as making the environment more meaningful, providing adequate light, etc., should be used. Making phone calls to friends or family members allows older adults to hear reassuring voices, which may be beneficial.

How well did you know this?
1
Not at all
2
3
4
5
Perfectly
42
Q
Which assessment finding of an older adult, who has a urinary tract infection, requires an immediate nursing intervention?
a.
Confusion
b.
Presbycusis
c.
Temperature of 97.9° F
d.
Death of a spouse 2 months ago
A

ANS: A
Confusion is a common manifestation in older adults with urinary tract infection; however, the cause requires further assessment. There may be another reason for the confusion. Confusion is not a normal finding in the older adult, even though it is commonly seen with concurrent infections. Difficulty hearing, presbycusis, is an expected finding in an older adult. Older adults tend to have lower core temperatures. Coping with the death of a spouse is a psychosocial concern to be addressed after the acute physiological concern in this case.

How well did you know this?
1
Not at all
2
3
4
5
Perfectly
43
Q

Which patient statement is the most reliable indicator that an older adult has the correct understanding of health promotion activities?
a.
“I need to increase my fat intake and limit protein.”
b.
“I still keep my dentist appointments even though I have partials now.”
c.
“I should discontinue my fitness club membership for safety reasons.”
d.
“I’m up-to-date on my immunizations, but at my age, I don’t need the influenza vaccine.”

A

ANS: B
General preventive measures for the nurse to recommend to older adults include keeping regular dental appointments to promote good oral hygiene, eating a low-fat, well-balanced diet, exercising regularly, and maintaining immunizations for seasonal influenza, tetanus, diphtheria and pertussis, shingles, and pneumococcal disease.

How well did you know this?
1
Not at all
2
3
4
5
Perfectly
44
Q
A 72-year-old woman was recently widowed. She worked as a teller at a bank for 40 years and has been retired for the past 5 years. She never learned how to drive. She lives in a rural area that does not have public transportation. Which psychosocial change does the nurse focus on as a priority?
a.
Sexuality
b.
Retirement
c.
Environment
d.
Social isolation
A

ANS: D
The highest priority at this time is the potential for social isolation. This woman does not know how to drive and lives in a rural community that does not have public transportation. All of these factors contribute to her social isolation. Other possible changes she may be going through right now include sexuality related to her advanced age and recent death of her spouse; however, this is not the priority at this time. She has been retired for 5 years, so this is also not an immediate need. She may eventually experience needs related to environment, but the data do not support this as an issue at this time.

How well did you know this?
1
Not at all
2
3
4
5
Perfectly
45
Q
A recently widowed older-adult patient is dehydrated and is admitted to the hospital for intravenous fluid replacement. During the evening shift, the patient becomes acutely confused. Which possible reversible causes will the nurse consider when assessing this patient? (Select all that apply.)
a.
Electrolyte imbalance
b.
Sensory deprivation
c.
Hypoglycemia
d.
Drug effects
e.
Dementia
A

ANS: A, B, C, D
Delirium, or a state of acute confusion, is a potentially reversible cognitive impairment that is often due to a physiological event. Physiological causes include electrolyte imbalances, untreated pain, infection, cerebral anoxia, hypoglycemia, medication effects, tumors, subdural hematomas, and cerebrovascular infarction or hemorrhage. Sometimes it is also caused by environmental factors such as sensory deprivation or overstimulation, unfamiliar surroundings, or sleep deprivation or psychosocial factors such as emotional distress. Dementia is a gradual, progressive, and irreversible cerebral dysfunction.

How well did you know this?
1
Not at all
2
3
4
5
Perfectly
46
Q

A nurse assesses clients at a family practice clinic for risk factors that could lead to dehydration. Which client is at greatest risk for dehydration?
a. A 36 year old who is presacribeidrblon.gc-toerm s/tetreoisdtherapy.
b. A 55 year old who recently received intravenous fluids.
ANS: C
c. A 76 year old who is cognitively impaired.
d. An 83 year old with congestive heart failure.

A

ANS: C
Older adults, because they have less total body water than younger adults, are at greater risk
for development of dehydration. Anyone who is cognitively impaired and cannot obtain fluids
independently or cannot make his or her need for fluids known is at high risk for dehydration. The client with heart failure has a risk for both fluid imbalances. Long-term steroids and recent IV fluid administration do not increase the risk of dehydration.

How well did you know this?
1
Not at all
2
3
4
5
Perfectly
47
Q

A nurse is caring for an older adult who exhibits dehydration-induced confusion. Which intervention by the nurse is best?

a. Measure intake and output every 4 hours
b. Assess client further for fall risk
c. Increase the IV flow rate to 250 mL/hr
d. Place the client in a high-Fowler’s position

A

ANS: B
Dehydration most frequently leads to poor perfusion and cerebral hypoxia, causing confusion. The client with dehydration is at risk for falls because of this confusion, orthostatic hypotension , dysrhythmias, and/or muscle weakness. The nurse’s best response is to do a more thorough evaluation of the client’s risk for falls. Measuring intake and output may need to occur more frequently than every 4 hours, but does not address critical need. The nurse would not adjust the IV flow rate without a prescription or standing protocol. For an older adult, this rapid an infusion rate could lead to fluid overload. Sitting the client in a high-Fowler’s position may or may not be comfortable but still does not address the most important issue which is safety.

How well did you know this?
1
Not at all
2
3
4
5
Perfectly
48
Q

After teaching a client who is being treated for dehydration, a nurse assesses the client’s understanding. Which statement indicates that the client correctly understood the teaching?

a. “I must drink a quart (liter) of water each day”
b. “I will weigh myself each morning before I eat or drink”
c. “I will use a salt substitute when making and eating my meals”
d. “I will not drink liquids after 6 p.m. so I won’t have to get up at night”

A

ANS: B
One liter of water weighs 1 kg; therefore, a change in body weight is a good measure of excess fluid loss or fluid retention. Weight loss greater than 0.5 lb (0.2 kg) daily is indicative of excess fluid loss. One liter of fluid a day is insufficient. A salt substitute is not related to dehydration. Clients may want to limit fluids after dinner so they won’t have to get up, but this does not address dehydration if the patient drinks the recommended amount of fluid during the earlier part of the day.

How well did you know this?
1
Not at all
2
3
4
5
Perfectly
49
Q

A nurse is assessing clients on a medical surgical unit. Which adult client does the nurse identify as being at greatest risk for insensible water loss?

a. Client taking furosemide
b. Anxious client with tachypnea
c. Client who is on fluid restrictions
d. Client who is constipated with abdominal pain

A

ANS: B
Insensible water loss is water loss through the skin, lungs and stool. Clients at risk for insensible water loss include those being mechanically ventilated, those with rapid respirations, and those undergoing continuous GI suctioning. Clients who have thyroid crisis, trauma burns, states of extreme stress, and fever are also at increased risk. The client taking furosemide will have increased fluid loss, but not insensible water loss. The other two clients on a fluid and with constipation are not at risk for insensible fluid loss.

How well did you know this?
1
Not at all
2
3
4
5
Perfectly
50
Q

A nurse is evaluating a client who is being treated for dehydration. Which assessment result does the nurse correlate with a therapeutic response to the treatment plan?

a. Increased respiratory rate from 12 to 22 breaths/min
b. Decreased skin turgor on the client’s posterior hand and forehead
c. Increased urine specific gravity from 1.012 to 1.030 g/mL
d. Decreased orthostatic changes when standing

A

ANS: D
The focus of management for clients with dehydration is to increase fluid volumes to normal. When blood volume is normal, orthostatic blood pressure and pulse changes will not occur. This assessment finding shows a therapeutic response to treatment. Increased respirations, decreased skin turgor, and higher urine specific gravity all are indicators of continuing dehydration.

How well did you know this?
1
Not at all
2
3
4
5
Perfectly
51
Q

After teaching a client who is prescribed a restricted sodium diet, a nurse assesses the client’s understanding. Which food choice for lunch indicates that the client correctly understood the teaching?

a. Slices of smoked ham with potato salad
b. Bowl of tomato soup with a grilled cheese
c. Salami and cheese on whole-wheat crackers
d. Grilled chicken breast with glazed carrots

A

ANS: D
Clients on restricted sodium diets generally avoid processes, smoked, and pickled foods and those with sauces and other condiments. Foods lowest in sodium include fish, poultry, and fresh produce. The ham, tomato soup, salami and crackers are usually high in sodium.

How well did you know this?
1
Not at all
2
3
4
5
Perfectly
52
Q

A nurse is assessing clients for fluid and electrolyte imbalances. Which client will the nurse assess first for potential hyponatremia?

a. A 34 year old who is NPO and receiving rapid intravenous D5W infusions
b. A 50 year old with an infection who is prescribed a sulfonamide antibiotic
c. A 67 year old who is experiencing pain and is prescribed ibuprofen
d. A 73 year old who is experiencing tachycardia who is receiving digoxin

A

ANS: A
Dextrose 5% in water (D5W) contains no electrolytes. The dextrose is rapidly metabolized when infused, leaving the solution hypotonic. Aggressive ingestion (or infusion) of hypotonic solutions can lead to hyponatremia. Because the client is not taking any food or fluids by mouth, normal sodium excretion can also lead to hyponatremia. The sulfonamide antibiotic, ibuprofen, and digoxin will not put a client at risk for hyponatremia.

How well did you know this?
1
Not at all
2
3
4
5
Perfectly
53
Q

A nurse teaches a client who is at risk for hyponatremia. Which statement does the nurse include in this client’s teaching?

a. “Have your spouse watch you for irritability and anxiety”
b. “Notify the clinic if you notice muscle twitching”
c. “Call your primary health care provider for diarrhea”
d. “Bake or grill your meat rather than frying it”

A

ANS: C
One sign of hyponatremia is diarrhea due to increased intestinal motility. The client would be taught to call the primary health care provider if this is noticed. Irritability and anxiety are common neurological signs of hypokalemia. Muscle twitching is related to hypernatremia. Cooking methods are not a cause of hypernatremia.

How well did you know this?
1
Not at all
2
3
4
5
Perfectly
54
Q

A nurse is caring for a client who has the following lab results: potassium 2.4 mEq/L (2.4 mmol/L), magnesium 1.8 mEq/L (0.74 mmol/L), calcium 8.5 mEq/L (2.13 mmol/L), and sodium 144 mEq/L (144 mmol/L). Which assessment does the nurse complete first?

a. Depth of respirations
b. Bowel sounds
c. Grip strength
d. Electrocardiography

A

ANS: A
A client with a low serum potassium level may exhibit hypoactive bowel sounds, cardiac dysrhythmias, and muscle weakness resulting in shallow respirations and decreased handgrips. The nurse would assess the client’s respiratory status first to ensure that respirations are sufficient. The respiratory assessment would include rate and depth of respirations, respiratory effort, and oxygen saturation. The other assessments are important but are secondary to the client’s respiratory status.

How well did you know this?
1
Not at all
2
3
4
5
Perfectly
55
Q

A nurse cares for a client who has a serum potassium of 6.5 mWq/L (6.5 mmol/L) and is exhibiting cardiovascular changes. Which intervention will the nurse implement first?

a. Prepare to administer patiromer by mouth
b. Provide a healthy heart, low potassium diet
c. Prepare to administer dextrose 20% and 10 units of regular insulin IV push
d. Prepare the client for hemodialysis treatment

A

ANS: C
A client with a critically high serum potassium level and cardiac changes would be treated immediately to reduce the extracellular potassium level. Potassium movement into the cells is enhanced by insulin by increasing the activity of sodium potassium pumps. Insulin will decreased both serum potassium and glucose levels and therefore would be administered with dextrose to prevent hypoglycemia. Patiromer may be ordered, but this therapy may take hours to reduce potassium levels.. Dialysis may also be needed, but this treatment will take much longer to implement and is not the first intervention the nurse would implement. Decreasing potassium intake may help prevent hyperkalemia in the future but will not decrease the client’s current potassium level.

How well did you know this?
1
Not at all
2
3
4
5
Perfectly
56
Q

The nurse is caring for a client who has fluid overload. What action by the nurse takes
priority?
a. Administer high-ceiling (loop) diuretics.
abirb.com/test
b. Assess the client’s lung sounds every 2 hours.
c. Place a pressure-relieving overlay on the mattress.
d. Weigh the client daily at the same time on the same scale.

A

ANS: B
All interventions are appropriate for the client who is overhydrated. However, client safety is the priority. A client with fluid overload can easily go into pulmonary edema, which can be life threatening. The nurse would closely monitor the client’s respiratory status.

How well did you know this?
1
Not at all
2
3
4
5
Perfectly
57
Q

A nurse is assessing a client with hypokalemia, and notes that the client’s handgrip strength
has diminished since the previous assessment 1 hour ago. What action does the nurse take
first?
a. Assess the client’s respiratory rate, rhythm, and depth.
b. Measure the client’s pulse and blood pressure.
c. Document findings and monitor the client.
d. Call the health care primary health care provider.

A

ANS: A
In a client with hypokalemia, progressive skeletal muscle weakness is associated with
increasing severity of hypokalemia. The most life-threatening complication of hypokalemia is respiratory insufficiency. It is imperative for the nurse to perform a respiratory assessment
first to make sure that the client is not in immediate jeopardy. Cardiac dysrhythmias are also
associated with hypokalemia. The client’s pulse and blood pressure would be assessed after
assessing respiratory status. Next, the nurse would call the health care primary health care provider to obtain orders for potassium replacement. Documenting findings and continuing to monitor the client would occur during and after potassium replacement therapy.

How well did you know this?
1
Not at all
2
3
4
5
Perfectly
58
Q

A new nurse is preparing to administer IV potassium to a client with hypokalemia. What action indicates the nurse needs to review this procedure?

a. Notifies the pharmacy of the IV potassium order.
b. Assesses the client’s IV site every hour during infusion.
c. Sets the IV pump to deliver 30 mEq of potassium an hour.
d. Double-checks the IV bag against the order with the precepting nurse.

A

ANS: C
IV potassium should not be infused at a rate exceeding 20mEq/hr under any circumstances. This action shows a need for further knowledge. The other actions are acceptable for this high-alert drug.

How well did you know this?
1
Not at all
2
3
4
5
Perfectly
59
Q

A nurse is caring for a client with hypocalcemia. Which action by the nurse shows poor
understanding of this condition?
a. Assesses the client’s Chvostek and Trousseau sign.
b. Keeps the client’s room quiet and dimly lit.
c. Moves the client carefully to avoid fracturing bones.
d. Administers bisphosphonates as prescribed

A

ANS: D
Bisphosphonates are used to treat hypercalcemia. The Chvostek and Trousseau signs are used to assess for hypocalcemia. Keeping the client in a low stimulus environment is important because the excitable nervous system cells are overstimulated. Long-standing hypocalcemia can cause fragile, brittle bones which can be fractured.

How well did you know this?
1
Not at all
2
3
4
5
Perfectly
60
Q

A nurse is caring for a client who has a serum calcium level of 14 mg/dL (3.5 mmol/L).
Which primary health care provider order does the nurse implement first?
a. Encourage oral fluid intake.
b. Connect the client to a cardiac monitor.
c. Assess urinary output.
d. Administer oral calcitonin.

A

ANS: B
This client has hypercalcemia. Elevated serum calcium levels can decrease cardiac output and
cause cardiac dysrhythmias. Connecting the client to a cardiac monitor is a priority to assess
for lethal cardiac changes. Encouraging oral fluids, assessing urine output, and administering
calcitonin are treatments for hypercalcemia, but are not the highest priority

How well did you know this?
1
Not at all
2
3
4
5
Perfectly
61
Q

A nurse is caring for an older adult client who is admitted with moderate dehydration. Which intervention will the nurse implement to prevent injury while in the hospital?

a. Ask family members to speak quietly to keep the client calm.
b. Assess urine color, amount, and specific gravity each day.
c. Encourage the client to drink at least 1 L of fluids each shift.
d. Dangle the client on the bedside before ambulating.

A

ANS: D
An older adult with moderate dehydration may experience orthostatic hypotension. The client needstodangleonthebedsidaebefiorrbea.mcboulmatin/gt.eAslthtoughdehydrationinanolderadultmay cause confusion, speaking quietly will not help the client remain calm or decrease confusion. Assessing the client’s urine may assist with the diagnosis of dehydration but would not
prevent injury. Clients are encouraged to drink fluids, but 1 L of fluid each shift for an older
adult may cause respiratory distress and symptoms of fluid overload, especially if the client has heart failure or renal insufficiency.

How well did you know this?
1
Not at all
2
3
4
5
Perfectly
62
Q
A nurse assesses a client who is admitted for treatment of fluid overload. Which signs and
symptoms does the nurse expect to find? (Select all that apply.) 
a. Increased pulse rate 
b. Distended neck veins
c. Decreased blood pressure
d. Warm and pink skin
e. Skeletal muscle weakness
f. Visual disturbances
A

ANS: A,B,E,F
Signs and symptoms of fluid overload include increased pulse rate, distended neck veins,
increased blood pressure, pale and cool skin, skeletal muscle weakness, and visual disturbances. Decreased blood pressure would be seen in dehydration. Warm and pink skin is a normal finding.

How well did you know this?
1
Not at all
2
3
4
5
Perfectly
63
Q

A nurse assesses a client who is prescribed a medication that inhibits aldosterone secretion
and release. For which potential complications will the nurse assess? (Select all that apply.)
a. Urine output of 25 mL/hr
b. Serum potassium level of 5.4 mEq/L (5.4 mmol/L)
c. Urine specific gravity of 1.02 g/mL
d. Serum sodium level of 128 mEq/L (128 mmol/L)
e. Blood osmolality of 250 mOsm/kg (250 mmol/kg)

A

ANS: B,E
Aldosterone is a naturally occurring hormone of the mineralocorticoid type that increases the
reabsorption of water and sodium in the kidney at the same time that it promotes excretion of potassium. Any drug or condition that disrupts aldosterone secretion or release increases the
client’s risk for excessive water loss (increased urine output), increased potassium
reabsorption, decreased blood osmolality, and increased urine specific gravity. The client
would not be at risk for sodium imbalance.

How well did you know this?
1
Not at all
2
3
4
5
Perfectly
64
Q

A nurse is assessing a client who has an electrolyte imbalance related to renal failure. For which potential complications of this electrolyte imbalance does the nurse assess? (Select all that apply.)

a. Reports of palpitations
b. Slow, shallow respirations
c. Orthostatic hypotension
d. Paralytic ileus
e. Skeletal muscle weakness
f. Tall, peaked T waves on ECG

A

ANS: A,E,F
Electrolyte imbalances associated with acute renal failure include hyperkalemia. The nurse would assess for electrocardiogram changes, including tall, peaked T waves, reports of palpitations or “skipped beats,” diarrhea, and skeletal muscle weakness in clients with hyperkalemia. The other choices are potential complications of hypokalemia. Respiratory muscles may be affected with lethally high hyperkalemia.

How well did you know this?
1
Not at all
2
3
4
5
Perfectly
65
Q

A nurse is caring for clients with electrolyte imbalances on a medical-surgical unit. Which clinical signs and symptoms are correctly paired with the contributing electrolyte imbalance?
(Select all that apply.)
a. Hypokalemia—muscle weakness with respiratory depression
b. Hypermagnesemia—bradycardia and hypotension
c. Hyponatremia—decreased level of consciousness
d. Hypercalcemia—positive Trousseau and Chvostek signs
e. Hypomagnesemia—hyperactive deep tendon reflexes
f. Hypernatremia—weak peripheral pulses

A

ANS: A,B,C,E,F
Hypokalemia is associated with muscle weakness and respiratory depression. Hypermagnesemia manifests with bradycardia and hypotension. Hyponatremia can present
with decreased level of consciousness. Hypomagnesemia can be assessed through hyperactive deep tendon reflexes. Weak peripheral pulses are felt in hypernatremia. Positive Trousseau and Chvostek signs are seen in hypocalcemia.

How well did you know this?
1
Not at all
2
3
4
5
Perfectly
66
Q

After administering potassium chloride, a nurse evaluates the client’s response. Which signs and symptoms indicate that treatment is improving the client’s hypokalemia? (Select all that apply.)

a. Respiratory rate of 8 breaths/min
b. Absent deep tendon reflexes
c. Strong productive cough
d. Active bowel sounds
e. U waves present on the electrocardiogram (ECG)

A

ANS: C, D
A strong, productive cough indicates an increase in muscle strength and improved potassium imbalance. Active bowel sounds also indicate that treatment is working. A respiratory rate of 8 breaths/min, absent deep tendon reflexes, and U waves present on the ECG are all signs and symptoms of hypokalemia and do not demonstrate that treatment is working.

How well did you know this?
1
Not at all
2
3
4
5
Perfectly
67
Q

A nurse develops a plan of care for an older client who has a fluid overload. What interventions will the nurse include in this client’s care plan? (Select all that apply.)

a. Calculate pulse pressure with each blood pressure reading.
b. Assess skin turgor using the back of the client’s hand.
c. Assess for pitting edema in dependent body areas.
d. Monitor trends in the client’s daily weights
e. Assist the client to change positions frequently
f. Teach client and family how to read food labels for sodium

A

ANS: A,C,D,E,F
Appropriate interventions for the client who has overhydration include calculating the pulse
pressure with each BP reading as this is a sign of cardiovascular involvement, assessing for pitting edema in the client’s dependent body areas, monitoring trends in the client’s daily weight as fluid retention is not always visible, protecting the client’s skin by helping him or
her change positions, and teaching the client and family to read food labels some type of sodium restriction may be required at home. The nurse assesses skin turgor on the chest or forehead.

How well did you know this?
1
Not at all
2
3
4
5
Perfectly
68
Q

A nurse is caring for clients with electrolyte imbalances on a medical-surgical unit. Which common causes are correctly paired with the corresponding electrolyte imbalance? (Select all that apply.)

a. Hypomagnesemia—kidney failure
b. Hyperkalemia—salt substitutes
c. Hyponatremia—heart failure
d. Hypernatremia—hyperaldosteronism
e. Hypocalcemia—diarrhea
f. Hypokalemia—loop diuretics

A

ANS: B,C,D,E,F
Salt substitutes contain potassium and are a cause of hyperkalemia. Hyponatremia can be caused by heart failure with fluid overload. Hyperaldosterone is a cause of hypernatremia and diarrhea causes actual calcium deficits. Loop diuretics excrete potassium. Decreased kidney function is a cause of magnesium excess, not deficit.

How well did you know this?
1
Not at all
2
3
4
5
Perfectly
69
Q

A nurse is caring for several clients at risk for fluid imbalances. Which laboratory results are
paired with the correct potential imbalance? (Select all that apply.)
a. Sodium: 160 mEq/L (mmol/L): Overhydration
b. Potassium: 5.4 mEq/L (mmol/L): Dehydration
c. Osmolarity: 250 mOsm/L: Overhydration
d. Hematocrit: 68%: Dehydration
e. BUN: 39 mg/dL: Overhydration
f. Magnesium: 0.8 mg/dL: Dehydration

A

ANS: B,C,D,F
In dehydration, hemoconcentaratbioinrubsu.aclloy rmesu/ltseinshtigher levels of hemoglobin, hematocrit, serum osmolarity, glucose, protein, blood urea nitrogen, and electrolytes. The opposite is true of overhydration. The sodium level is high, indicating dehydration. The potassium level is high, also indicating possible dehydration. The osmolarity is low, indicating overhydration,
the hematocrit is high indicating dehydration, the BUN is high indicating dehydration, and the magnesium level is low, indicating possible dehydration and malnutrition from
diarrhea-causing diseases.

How well did you know this?
1
Not at all
2
3
4
5
Perfectly
70
Q

A nurse assesses a client with diabetes mellitus who is admitted with an acid–base imbalance. The client’s arterial blood gas values are pH 7.36, PaO2 98 mm Hg, PaCO2 33 mm Hg, and HCO3 18 mEq/L (18 mmol/L). Which sign or symptom does the nurse identify as an example of the client’s compensatory mechanisms?

a. Increased rate and depth of respirations
b. Increased urinary output
c. Increased thirst and hunger
d. Increased release of acids from the kidneys

A

ANS: A
This client has metabolic acidosis. The respiratory system compensates by increasing its activity and blowing off excess carbon dioxide. Increased urinary output, thirst, and hunger are signs and symptoms of hyperglycemia but are not compensatory mechanisms for acid–base imbalances. The kidneys do not release acids.

How well did you know this?
1
Not at all
2
3
4
5
Perfectly
71
Q

A nurse assesses a client who is experiencing an acid–base imbalance. The client’s arterial blood gas values are pH 7.2, PaO2 88 mmHg, PaCO2 38 mmHg, and HCO3 19mEq/L (19 mmol/L). Which assessment would the nurse perform first?

a. Cardiac rate and rhythm
b. Skin and mucous membranes
c. Musculoskeletal strength
d. Level of orientation

A

ANS: A
Early cardiovascular changes for a client experiencing moderate acidosis include increased heart rate and cardiac output. As the acidosis worsens, the heart rate decreases and electrocardiographic changes will be present. The nurse responds by performing a thorough cardiovascular assessment. Changes will occur in the integumentary system, musculoskeletal system, and neurological system, but assessing for the cardiovascular complications comes first.

How well did you know this?
1
Not at all
2
3
4
5
Perfectly
72
Q

A nurse assesses a client who is prescribed furosemide for hypertension. For which acid–base
imbalance does the nurse assess to prevent complications of this therapy?
a. Respiratory acidosis
b. Respiratory alkalosis
c. Metabolic acidosis
d. Metabolic alkalosis

A

ANS: D
Many diuretics, especially loop and thiazide diuretics, increase the excretion of hydrogen ions, leading to excess acid loss through the renal system. This situation is an actual acid deficit.

How well did you know this?
1
Not at all
2
3
4
5
Perfectly
73
Q

A nurse is caring for a client who is experiencing moderate metabolic alkalosis. What action
would the nurse take?
a. Monitor daily hemoglobin and hematocrit values
b. Administer furosemide intravenously.
c. Encourage the client to take deep breaths.
d. Teach the client fall prevention measures.

A

ANS: D
The most important nursing care for a client who is experiencing moderate metabolic alkalosis is providing client safety. Client’s with metabolic alkalosis have muscle weakness and are at risk for falling. The other nursing interventions are not appropriate for metabolic alkalosis.

How well did you know this?
1
Not at all
2
3
4
5
Perfectly
74
Q

A nurse is assessing a client who has acute pancreatitis and is at risk for an acid–base imbalance. For which manifestation of this acid–base imbalance would the nurse assess?

a. Agitation
b. Kussmaul respirations
c. Seizures
d. Positive Chvostek sign

A

ANS: B
The pancreas is a major site of bicarbonate production. Pancreatitis can cause metabolic acidosis through underproduction of bicarbonate ions. Signs and symptoms of acidosis include lethargy and Kussmaul respirations. Agitation, seizures, and a positive Chvostek sign are signs and symptoms of the electrolyte imbalances that accompany alkalosis.

How well did you know this?
1
Not at all
2
3
4
5
Perfectly
75
Q

A nurse assesses a client who is admitted with an acid–base imbalance. The client’s arterial blood gas values were pH 7.32, PaO2 85 mm Hg, PaCO2 34 mm Hg, and HCO3 16 mEq/L (16 mmol/L). The most recent blood gasses show a drop in the pH. What action does the nurse take next?

a. Assess client’s rate, rhythm, and depth of respiration.
b. Measure the client’s pulse and blood pressure.
c. Document the findings and continue to monitor.
d. Notify the primary health care provider.

A

ANS: A
Progressive skeletal muscle weakness is associated with increasing severity of acidosis. Muscle weakness can lead to severe respiratory insufficiency. Acidosis does lead to dysrhythmias (due to hyperkalemia), but these would best be assessed with cardiacmonitoring. Findings would be documented, but simply continuing to monitor is not sufficient. Before notifying the primary care provider, the nurse must have more data to report.

How well did you know this?
1
Not at all
2
3
4
5
Perfectly
76
Q

A nurse is caring for a client who has the following arterial blood values: pH 7.12, PaO2 56 mm Hg, PaCO2 65 mm Hg, and HCO3 22 mEq/L (22 mmol/L). Which clinical situation does the nurse correlate with these values?

a. Diabetic ketoacidosis in a person with emphysema
b. Bronchial obstruction related to aspiration of a hot dog
c. Anxiety-induced hyperventilation in an adolescent
d. Diarrhea for 36 hours in an older, frail woman

A

ANS: B
Arterial blood gas values indicate that the client has acidosis with normal levels of
bicarbonate, suggesting that the problem is not metabolic. Arterial concentrations of oxygen
and carbon dioxide are abnormal, with low oxygen and high carbon dioxide levels. Thus, this client has respiratory acidosis from inadequate gas exchange. The fact that the bicarbonate level is normal indicates that this is an acute respiratory problem rather than a chronic problem, because no renal compensation has occurred. The client who would have these ABG values is the one with the new onset of airway obstruction.

How well did you know this?
1
Not at all
2
3
4
5
Perfectly
77
Q

A nurse is caring for a client who has just experienced a 90-second tonic-clonic seizure. The client’s arterial blood gas values are pH 6.88, PaO2 50 mm Hg, PaCO2 60 mm Hg, and HCO3 22 mEq/L (22 mmol/L). What action would the nurse take first?

a. Apply oxygen by mask or nasal cannula.
b. Apply a paper bag over the client’s nose and mouth.
c. Administer 50 mL of sodium bicarbonate intravenously.
d. Administer 50 mL of 20% glucose and 20 units of regular insulin.

A

ANS: A
This client is severely hypoxic and needs oxygen. Now that the seizure has ended, the client can breathe again normally, so oxygen administration will rapidly increase the PaO2. Rebreathing carbon dioxide with a paper bag would make the acidosis worse. Bicarbonate is only indicated with extremely low pH and serum bicarbonate levels. Glucose and insulin are administered to decrease the high potassium levels associated with acidosis, but this situation should reverse itself with oxygen and breathing.

How well did you know this?
1
Not at all
2
3
4
5
Perfectly
78
Q

After teaching a client who was malnourishing and is being discharged, a nurse assesses the client’s understanding. Which statement indicates that the client correctly understood teaching to decrease risk for the development of metabolic acidosis?

a. “I will drink at least three glasses of milk each day.”
b. “I will eat three well-balanced meals and a snack daily.”
c. “I will not take pain medication and antihistamines together.”
d. “I will avoid salting my food when cooking or during meals.”

A

ANS: B
Starvation or a diet with too few carbohydrates can lead to metabolic acidosis by forcing cells metabolism. Eating sufficient calories from all food groups helps reduce this risk. Milk, taking pain medications with antihistamines, and salting food are not related.

How well did you know this?
1
Not at all
2
3
4
5
Perfectly
79
Q

A nurse evaluates the following arterial blood gas values in a client: pH 7.48, PaO2 98 mm
Hg, PaCO2 28 mm Hg, and HCO3 22 mEq/L (22 mmol/L). Which client condition does the nurse correlate with these results?
a. Diarrhea and vomiting for 36 hours
b. Anxiety-induced hyperventilation
c. Chronic obstructive pulmonary disease (COPD)
d. Diabetic ketoacidosis and emphysema

A

ANS: B
The elevated pH level indicates alkalosis. The bicarbonate level is normal, and so is the
oxygen partial pressure. Loss of carbon dioxide is the cause of the alkalosis, which would occur in response to hyperventilation. Diarrhea and vomiting would cause metabolic acidosis and COPD would lead to respiratory acidosis. The client with emphysema most likely would have combined metabolic acidosis on top of a mild, chronic respiratory acidosis.

How well did you know this?
1
Not at all
2
3
4
5
Perfectly
80
Q

After providing discharge teaching, a nurse assesses the client’s understanding regarding increased risk for metabolic alkalosis. Which statement indicates that the client needs additional teaching?

a. “I don’t drink milk because it gives me gas and diarrhea”
b. “I have been taking digoxin every day for the last 15 years.”
c. “I take sodium bicarbonate after every meal to prevent heartburn.”
d. “In hot weather, I sweat so much that I drink six glasses of water each day.”

A

ANS: C
Excessive oral ingestion of sodium bicarbonate and other bicarbonate-based antacids can cause metabolic alkalosis. Avoiding milk, taking digoxin, and sweating would not lead to increased risk of metabolic alkalosis.

How well did you know this?
1
Not at all
2
3
4
5
Perfectly
81
Q

A nurse is caring for a client who is experiencing excessive diarrhea. The client’s arterial blood gas values are pH 7.18, PaO2 98 mm Hg, PaCO2 45 mm Hg, and HCO3 16 mEq/L (16 mmol/L). Which primary health care provider order does the nurse expect to receive?

a. Furosemide 40 mg
b. Sodium bicarbonate
c. Mechanical ventilation
d. Indwelling urinary catheter

A

ANS: B
This client’s arterial blood gas values represent metabolic acidosis related to a loss of bicarbonate ions from diarrhea. The bicarbonate would be replaced to help restore this client’s acid–base balance as the pH is below 7.2 and the bicarbonate level is low. Furosemide would cause an increase in acid fluid and acid elimination via the urinary tract; although this may improve the client’s pH, the client has excessive diarrhea and cannot afford to lose more fluid. Mechanical ventilation is used to treat respiratory acidosis for clients who cannot keep their oxygen saturation at 90%, or who have respiratory muscle fatigue. Mechanical ventilation and an indwelling urinary catheter would not be prescribed for that client.

How well did you know this?
1
Not at all
2
3
4
5
Perfectly
82
Q

A nurse evaluates a client’s arterial blood gas values (ABGs): pH 7.30, PaO2 86 mm Hg, PaCO2 55 mm Hg, and HCO3 22 mEq/L (22 mmol/L). Which intervention does the nurse implement first?

a. Assess the airway.
b. Administer prescribed bronchodilators.
c. Provide oxygen.
d. Administer prescribed mucolytics.

A

ANS: A
All interventions are important for clients with respiratory acidosis; this is indicated by the ABGs. However, the priority is assessing and maintaining an airway. Without a patent airway, other interventions will not be helpful.

How well did you know this?
1
Not at all
2
3
4
5
Perfectly
83
Q

A nurse is planning care for a client who is hyperventilating. The client’s arterial blood gas
values are pH 7.52, PaO2 94 mm Hg, PaCO2 31 mm Hg, and HCO3 26 mEq/L (26 mmol/L). Which question would the nurse ask when developing this client’s plan of care?
a. “Do you take any over-the-counter medications?”
b. “You appear anxious. What is causing your distress?”
c. “Do you have a history of anxiety attacks?”
d. “You are breathing fast. Is this causing you to feel light-headed?”

A

ANS: B
The nurse would assist the client who is experiencing anxiety-induced respiratory alkalosis to identify causes of the anxiety. The other questions will not identify the cause of the acid–base imbalance. The other three questions are also yes/no and close-ended.

84
Q

A diabetic client becomes septic after a bowel resection and is having problems with
respiratory distress. The nurse reviews the labs and finds the following ABG results: pH 7.50,
PaCO2 30, HCO3 : 24, and PaO2 68. What does the nurse recognize as the primary factor
causing this the acid–base imbalance?
a. Atelectasis due to respiratory muscle fatigue
b. Hyperventilation due to poor oxygenation
c. Hypoventilation due to morphine PCA
d. Kussmaul respirations due to glucose of 102 mg/dL (5.7 mmol/L)

A

ANS: B
The ABG results indicate respiratory alkalosis. The client has low oxygenation as indicated by
cause respiratory acidosis with a low pH and high PaCO2. Kussmaul respirations are
low partial pressure of arterial oxygen causing a compensatory mechanism of increased
respirations and hyperventilation. Respiratory muscle fatigue and hypoventilation would characterized by deep labored breathing and are a compensatory mechanism to metabolic acidosis, not hypoxemia or alkalosis.

85
Q

A nurse is planning interventions that regulate acid–base balance to ensure that the pH of a client’s blood remains within the normal range. Which abnormal physiologic functions may occur if the client experiences an acid-base imbalance?(Select all that apply.)

a. Reduction in the function of hormones
b. Fluid and electrolyte imbalances
c. Increase in the function of selected enzymes
d. Excitable cardiac muscle membranes
e. Increase in the effectiveness of many drugs
f. Changes in GI tract excitability

A

ANS: A,B,D,F
Acid–base imbalances interfere with normal physiology, including reducing the function of and GI tract to be more or less excitable, and decreasing the effectiveness of many drugs.

86
Q

A nurse assesses a client who is experiencing an acid–base imbalance. The client’s arterial blood gas values are pH 7.32, PaO2 94 mm Hg, PaCO2 34 mm Hg, and HCO3 18 mEq/L (18mmol/L). For which clinical signs and symptoms would the nurse assess? (Select all that

apply. )
a. Reduced deep tendon reflexes
b. Drowsiness
c. Increased respiratory rate
d. Decreased urinary output
e. Positive Trousseau sign
f. Flaccid paralysis

A

ANS: A,B,C
Metabolic acidosis causes neuromuscular changes, including reduced muscle tone and deep
tendon reflexes. Clients usually present with lethargy and drowsiness. The respiratory system
will attempt to compensate for the metabolic acidosis; therefore, respirations will increase rate
and depth. Flaccid paralysis can occur. A positive Trousseau sign is associated with alkalosis.
Decreased urine output is not a sign of metabolic acidosis.

87
Q

A nurse is assessing clients who are at risk for acid–base imbalance. Which clients are
correctly paired with the acid–base imbalance? (Select all that apply.)
a. Metabolic alkalosis—young adult who is prescribed intravenous morphine sulfate
for pain
b. Metabolic acidosis—older adult who is following a carbohydrate-free diet
c. Respiratory alkalosis—client on mechanical ventilation at a rate of 28 breaths/min
d. Respiratory acidosis—postoperative client who received 6 units of packed red blood cells
e. Metabolic alkalosis—older client prescribed antacids for gastroesophageal reflux
disease

A

ANS: B,C,E
Respiratory acidosis often occurs as the result of underventilation. The client who is taking opioids, especially IV opioids, is at risk for respiratory depression and respiratory acidosis. One cause of metabolic acidosis is a strict low-calorie diet or one that is low in carbohydrate content. Such a diet increases the rate of fat catabolism and results in the formation of excessive ketoacids. A ventilator set at a high respiratory rate or tidal volume will cause the client to lose too much carbon dioxide, leading to an acid deficit and respiratory alkalosis.Citrate is a substance used as a preservative in blood products. It is not only a base, but also a
precursor for bicarbonate. Multiple units of packed red blood cells could cause metabolic alkalosis. Sodium bicarbonate antacids may increase the risk of metabolic alkalosis.

88
Q

A nurse assesses a client who is receiving total parenteral nutrition. For which adverse effects related to an acid–base imbalance would the nurse assess? (Select all that apply.)

a. Positive Chvostek sign
b. Elevated blood pressure
c. Bradycardia
d. Increased muscle strength
e. Anxiety and irritability
f. Tetany

A

ANS: A,E
A client receiving total parenteral nutrition is at risk for metabolic alkalosis. Signs and symptoms of metabolic alkalosis include positive Chvostek sign, normal or low blood pressure, increased heart rate, skeletal muscle weakness, possible tetany and seizures, and anxiety and irritability.

89
Q

A nurse is planning care for a client who is lethargic and confused. The client’s arterial blood gas values are pH 7.30, PaO2 96 mm Hg, PaCO2 43 mm Hg, and HCO3 19 mEq/L (19
mmol/L). Which questions would the nurse ask the client and spouse when developing the plan of care? (Select all that apply.)
a. “Are you taking any antacid medications?”
b. “Is your spouse’s current behavior typical?”
c. “Do you drink any alcoholic beverages?”
d. “Have you been participating in strenuous activity?”
e. “Are you experiencing any shortness of breath?”

A

ANS: B,C,D
This client’s symptoms of lethargy and confusion are related to a state of metabolic acidosis. The nurse would ask the client’s spouse or family members if the client’s behavior is typical for him or her, and establish a baseline for comparison with later assessment findings. The nurse would also assess for alcohol intake because alcohol can cause metabolic acidosis. Excessive and strenuous activity can lead to overproduction of hydrogen ions. The other options are not causes of metabolic acidosis.

90
Q

A patient’s serum osmolality is 305 mOsm/kg. Which term describes this patient’s body fluid osmolality?

a. Iso-osmolar
b. Hypo-osmolar
c. Hyper-osmolar
d. Isotonic

A

ANS: C

Normal osmolality is 275 to 295 mOsm/kg. This patient is therefore hyper-osmolar.

91
Q

A patient is admitted after experiencing vomiting and diarrhea for several days. The provider orders intravenous lactated Ringer’s solution. The nurse understands that this fluid is given for which purpose?

a. To increase interstitial and intracellular hydration
b. To maintain plasma volume over time
c. To pull water from the interstitial space into the extracellular fluid
d. To replace water and electrolytes

A

ANS: D
Lactated Ringer’s solution is an isotonic solution and is used to replace water and electrolytes and is often used to replace gastrointestinal losses. Hypotonic fluids increase interstitial and intracellular hydration. Colloidal solutions are used to maintain plasma volume over time. Hypertonic solutions pull water from the interstitial space into the extracellular fluid.

92
Q

A patient is being treated for shock after a motor vehicle accident. The provider orders 6% dextran 75 to be given intravenously. The nurse should expect which outcome as the result of this infusion?

a. Decreased urine output
b. Improved blood oxygenation
c. Increased interstitial fluid
d. Stabilization of heart rate and blood pressure

A

ANS: D
6% Dextran 75 is a high–molecular-weight colloidal solution and is used to treat shock from hemorrhage, burns, or trauma. Colloids are plasma expanders, and the end result is an improvement in heart rate (decreased) and blood pressure (increased). Plasma expanders will result in an increase in urine output. Blood oxygenation is not affected, and colloids do not increase the amount of interstitial fluid.

93
Q

The nurse is caring for a patient who weighs 75 kg. The patient has intravenous (IV) fluids infusing at a rate of 50 mL/h and has consumed 100 mL of fluids orally in the past 24 hours. Which action will the nurse take?

a. Contact the provider to ask about increasing the IV rate to 90 mL/h.
b. Discuss with the provider the need to increase the IV rate to 150 mL/h.
c. Encourage the patient to drink more water so the IV can be discontinued.
d. Instruct the patient to drink 250 mL of water every 8 hours.

A

ANS: A
The recommended daily fluid intake for adults is 30 to 40 mL/kg/day. This patient should
have a minimum of 2250 mL/day and is currently receiving 1200 mL IV plus 100 mL orally
for a total of 1300 mL. Increasing the IV rate to 90 mL/h would give the patient 2160 mL. If
the patient continues to take oral fluids, the amount of 2250 mL can be met. A rate of 150
mL/h would give the patient 3600 mL/day, which exceeds the recommended amount. Since
this patient is not taking fluids well and is not receiving adequate IV fluids, encouraging an
increased fluid intake is not indicated. Even if the patient drank 250 mL of water every 8
hours, the amount would not be sufficient.

94
Q

The nurse is caring for a patient who has a heart rate of 98 beats per minute and a blood pressure of 82/58 mm Hg. The patient is lethargic, is complaining of muscle weakness, and has had gastroenteritis for several days. Based on this patient’s vital signs, which sodium value would the nurse expect?

a. 126 mEq/L
b. 140 mEq/L
c. 145 mEq/L
d. 158 mEq/L

A

ANS: A
Patients who are hyponatremic will have tachycardia and hypotension along with lethargy and muscle weakness. The normal range for serum sodium is 135 to 145 mEq/L; a serum sodium level of 126 mEq/L would be considered hyponatremic.

95
Q

The nurse is caring for a patient who has had severe vomiting. The patient’s serum sodium level is 130 mEq/L. The nurse will expect the patient’s provider to order which treatment?

a. Diuretic therapy
b. Intravenous hypertonic 5% saline
c. Intravenous normal saline 0.9%
d. Oral sodium supplements

A

ANS: C
Patients with hyponatremia may be treated with oral sodium supplements if the patient is able or if the deficit is mild. This patient is vomiting and would not be able to take supplements easily. For a serum sodium level between 125 and 135 mEq/L, normal saline may increase sodium content in vascular fluid. Hypertonic saline is used for severe hyponatremia with a serum sodium <120 mEq/L. Diuretics would further deplete sodium and fluid volume in a patient already likely to be dehydrated from severe vomiting.

96
Q

The nurse is caring for a newly admitted patient who has severe gastroenteritis. The patient’s electrolytes reveal a serum sodium level of 140 mEq/L and a serum potassium level of 3.5 mEq/L. The nurse receives an order for intravenous 5% dextrose and normal saline with 20 mEq/L potassium chloride to infuse at 125 mL/h. Which action is necessary prior to administering this fluid?

a. Evaluate the patient’s urine output.
b. Contact the provider to order arterial blood gases.
c. Request an order for an initial potassium bolus.
d. Suggest a diet low in sodium and potassium.

A

ANS: A
If the patient is receiving potassium and the urine output is <25 mL/h or <600 mL/d, potassium accumulation may occur. Patients with a low urine output should not receive IV potassium. Arterial blood gases are not necessary prior to IV potassium administration. Potassium should never be given as a bolus. Patients should be put on a potassium-enriched diet when foods are tolerated.

97
Q

A patient who is being treated for dehydration is receiving 5% dextrose and 0.45% normal saline with 20 mEq/L potassium chloride at a rate of 125 mL/h. The nurse assuming care for the patient reviews the patient’s serum electrolytes and notes a serum sodium level of 140 mEq/L and a serum potassium level of 3.6 mEq/L. The patient had a urine output of 250 mL during the last 12-hour shift. Which action will the nurse take?

a. Contact the patient’s provider to discuss increasing the potassium chloride to 40 mEq/L.
b. Continue the intravenous fluids as ordered and reassess the patient frequently.
c. Notify the provider and discuss increasing the rate of fluids to 200 mL/h.
d. Stop the intravenous fluids and notify the provider of the assessment findings.

A

ANS: D
The patient’s potassium level is within normal limits, but the urine output is decreased, so the patient should not be receiving IV potassium. The nurse should stop the IV and report the findings to the provider. The patient does not need an increase in potassium. The patient needs more fluids but not with potassium.

98
Q

A patient has a serum potassium level of 2.7 mEq/L. The patient’s provider has determined that the patient will need 200 mEq of potassium to replace serum losses. How will the nurse caring for this patient expect to administer the potassium?

a. As a single-dose 200 mEq oral tablet
b. As an intravenous bolus over 15 to 20 minutes
c. In an intravenous solution at a maximum rate of 10 mEq/h
d. In an intravenous solution at a rate of 45 mEq/h

A

ANS: C
Potassium chloride should be given intravenously when hypokalemia is severe, so this patient should receive IV potassium chloride. Potassium should never be given as a bolus and should be administered slowly. The maximum infusion rate for adults with a serum potassium level greater than 2.5 mEq/L is 10 mEq/h or 200 mEq/24 hours.

99
Q
A patient is taking a loop diuretic and reports anorexia and fatigue. The nurse suspects which
electrolyte imbalance in this patient? 
a. Hypercalcemia
b. Hypocalcemia
c. Hyperkalemia
d. Hypokalemia
A

ANS: D
Loop diuretics cause the body to lose potassium. Patients who take loop and thiazide diuretics should be monitored for hypokalemia.

100
Q

The nurse is caring for a patient whose serum sodium level is 140 mEq/L and serum potassium level is 5.4 mEq/L. The nurse will contact the patient’s provider to discuss an order for:

a. a low-potassium diet.
b. intravenous sodium bicarbonate.
c. Sodium polystyrene sulfonate (Kayexalate).
d. salt substitutes.

A

ANS: A
Mild hyperkalemia may be treated with dietary restriction of potassium-rich foods. The patient’s sodium level is normal, so sodium bicarbonate is not indicated. Kayexalate is used for severe hyperkalemia. Salt substitutes contain potassium and would only compound the hyperkalemia.

101
Q

The provider has ordered sodium polystyrene sulfonate (Kayexalate) to be administered to a patient. The nurse caring for this patient would expect which serum electrolyte values prior to administration of this therapy?

a. Sodium 125 mEq/L and potassium 2.5 mEq/L
b. Sodium 150 mEq/L and potassium 3.6 mEq/L
c. Sodium 135 mEq/L and potassium 6.9 mEq/L
d. Sodium 148 mEq/L and potassium 5.5 mEq/L

A

ANS: C
Severe hyperkalemia, with a potassium level of 6.9 mEq/L, requires aggressive treatment to increase the body’s excretion of potassium. Kayexalate is a potassium binder used to treat severe hyperkalemia. The normal range for serum potassium is 3.5 to 5.5 mEq/L, so patients with the other potassium levels would not be treated aggressively or would need potassium supplementation.

102
Q

The nurse is caring for a patient who is receiving oral potassium chloride supplements. The nurse notes that the patient has a heart rate of 120 beats per minute and has had a urine output of 200 mL in the past 12 hours. The patient reports abdominal cramping. Which action will the nurse take?

a. Contact the provider to request an order for serum electrolytes.
b. Encourage the patient to consume less fluids.
c. Report symptoms of hyperchloremia to the provider.
d. Request an order to increase the patient’s potassium dose.

A

ANS: A
Oliguria, tachycardia, and abdominal cramping are signs of hyperkalemia, so the nurse should request an order for serum electrolytes. This patient should increase fluid intake. The patient is not exhibiting signs of hyperchloremia; the patient is showing signs of hyperkalemia, and an increased potassium dose is not indicated.

103
Q

A patient asks the nurse about taking calcium supplements to avoid hypocalcemia. The nurse will suggest that the patient follow which instruction?

a. Take a calcium and vitamin D combination supplement.
b. Take calcium along with phosphorus to improve absorption.
c. Take calcium with antacids to reduce stomach upset.
d. Use aspirin instead of acetaminophen when taking calcium.

A

ANS: A
Vitamin D enhances the absorption of calcium in the body. Calcium and phosphorus have an inverse relationship—an increased level of one mineral decreases the level of the other, and they would bind in the gastrointestinal tract and not be absorbed if taken at the same time. Antacids can contain magnesium, which can promote calcium loss. Aspirin can alter vitamin D levels and interfere with calcium absorption.

104
Q

The nurse is caring for a newly admitted patient who will receive digoxin to treat a cardiac dysrhythmia. The patient takes hydrochlorothiazide (HydroDIURIL) and reports regular use of over-the-counter laxatives. Before administering the first dose of digoxin, the nurse will review the patient’s electrolytes with careful attention to the levels of which electrolytes?

a. Calcium and magnesium
b. Sodium and calcium
c. Potassium and chloride
d. Potassium and magnesium

A

ANS: D
Hypomagnesemia, like hypokalemia, enhances the action of digitalis and causes digitalis toxicity. Laxatives and diuretics can deplete both of these electrolytes.

105
Q

The nurse is administering intravenous fluids to a patient who is dehydrated. On the second day of care, the patient’s weight is increased by 2.25 pounds. The nurse would expect that the patient’s fluid intake has

a. equaled urine output.
b. exceeded urine output by 1 L.
c. exceeded urine output by 2.5 L.
d. exceeded urine output by 3 L.

A

ANS: B

A weight gain of 1 kg, or 2.2 to 2.5 lb, is equivalent to 1 L of fluid.

106
Q

The nurse is caring for a patient who is receiving isotonic intravenous (IV) fluids at an infusion rate of 125 mL/h. The nurse performs an assessment and notes a heart rate of 102 beats per minute, a blood pressure of 160/85 mm Hg, and crackles auscultated in both lungs. Which action will the nurse take?

a. Decrease the IV fluid rate and notify the provider.
b. Increase the IV fluid rate and notify the provider.
c. Request an order for a colloidal IV solution.
d. Request an order for a hypertonic IV solution.

A

ANS: A
The patient shows signs of fluid volume excess, so the nurse should slow the IV fluid rate and notify the provider. Increasing the rate would compound the problem. Colloidal and hypertonic fluids would pull more fluids into the intravascular space and compound the problem.

107
Q

The nurse is preparing to administer digoxin to a patient who is newly admitted to the intensive care unit. The nurse reviews the patient’s admission electrolytes and notes a serum potassium level of 2.9 mEq/L. Which action by the nurse is correct?

a. Administer the digoxin and monitor the patient’s electrocardiogram closely.
b. Hold the digoxin dose and notify the provider of the patient’s lab values.
c. Request an order for an intravenous bolus of potassium.
d. Request an order for oral potassium supplements.

A

ANS: B
Hypokalemia increases the risk for digoxin toxicity, so the nurse should hold the dose and notify the provider. Potassium should never be given as an IV bolus. Oral supplements are not used when hypokalemia is severe.

108
Q

The nurse is performing an assessment on a patient brought to the emergency department for
treatment for dehydration. The nurse assesses a respiratory rate of 26 breaths/minute, a heart rate of 110 beats/minute, a blood pressure of 86/50 mm Hg, and a temperature of 39.5° C. The patient becomes dizzy when transferred from the wheelchair to a bed. The nurse notes cool, clammy skin. Which diagnosis does the nurse suspect?
a. Fluid volume deficit (FVD)
b. Fluid volume excess (FVE)
c. Mild extracellular fluid (ECF) deficit
d. Renal failure

A

ANS: A
Patients with FVD will exhibit elevated temperature, tachycardia, tachypnea, hypotension, orthostatic hypotension, and cool, clammy skin. Patients with FVE will have bounding pulses, elevated blood pressure, dyspnea, and crackles. Mild ECF deficit causes thirst. Renal failure generally leads to FVE.

109
Q

The nurse is caring for a patient who will receive 10% calcium gluconate to treat a serum potassium level of 5.9 mEq/L. The nurse performs a drug history prior to beginning the infusion. Which drug taken by the patient would cause concern?

a. Digitalis
b. Hydrochlorothiazide
c. Hydrocortisone
d. Vitamin D

A

ANS: A
Calcium gluconate is given to treat hyperkalemia in order to decrease irritability of the myocardium. When administered to a patient taking digitalis, it can cause digitalis toxicity. The other drugs may affect potassium levels but are not a cause for concern with calcium gluconate.

110
Q

A patient is admitted with orthopnea, cough, pulmonary crackles, and peripheral edema. The patient’s urine specific gravity is 1.002. The nurse will expect this patient’s provider to order which treatment?

a. Diuretics
b. Colloidal IV fluids
c. Hypertonic IV fluids
d. Hypotonic IV fluids

A

ANS: A
This patient has signs of fluid volume excess. Urine specific gravity levels less than 1.010 g/mL indicate dilute urine and excess fluid. Diuretics are prescribed to reduce fluid overload.

111
Q

The nurse is caring for a patient who will receive intravenous calcium gluconate. Which nursing actions are appropriate when giving this solution? (Select all that apply.)

a. Administering through a central line
b. Review the patients medication record to see if they are receiving digitalis
c. Giving as a rapid intravenous bolus
d. Mixing in a solution containing sodium bicarbonate
e. Monitoring the patient’s electrocardiogram (ECG)
f. Reporting a serum calcium level of >2.5 mEq/L

A

ANS: B, E, F
Calcium gluconate has the same action on the heart as digitalis and combined use can place the patient at risk for digitalis toxicity. Hypercalcemia can cause ECG changes. A serum calcium level greater than 2.5 mEq/L indicates hypercalcemia and therefore should be reported. Calcium does not require infusion through a central line and should not be given as a rapid IV bolus. Calcium should not be added to a solution containing bicarbonate, because rapid precipitation occurs.

112
Q

A patient is suspected of having severe hypocalcemia. While waiting for the patient’s serum electrolyte results, the nurse will assess for which symptoms? (Select all that apply.)

a. Laryngeal spasms
b. Fatigue
c. Muscle weakness
d. Nausea and vomiting
e. Hyperactive deep tendon reflexes
f. Twitching of the mouth

A

ANS: A, E, F
Patients who have hypocalcemia will exhibit laryngeal spasms, hyperactive deep tendon reflexes, and twitching of the mouth. The other symptoms are not characteristic of hypocalcemia.

113
Q
A patient is experiencing dehydration. While planning care, the nurse considers that the majority of the patient’s total water volume exists in with compartment?
a.
Intracellular
b.
Extracellular
c.
Intravascular
d.
Transcellular
A

ANS: A
Intracellular (inside the cells) fluid accounts for approximately two thirds of total body water. Extracellular (outside the cells) is approximately one third of the total body water. Intravascular fluid (liquid portion of the blood) and transcellular fluid are two major divisions of the extracellular compartment.

114
Q
The nurse is teaching about the process of passively moving water from an area of lower particle concentration to an area of higher particle concentration. Which process is the nurse describing?
a.
Osmosis
b.
Filtration
c.
Diffusion
d.
Active transport
A

ANS: A
The process of moving water from an area of low particle concentration to an area of higher particle concentration is known as osmosis. Filtration is mediated by fluid pressure from an area of higher pressure to an area of lower pressure. Diffusion is passive movement of electrolytes or other particles down the concentration gradient (from areas of higher concentration to areas of lower concentration). Active transport requires energy in the form of adenosine triphosphate (ATP) to move electrolytes across cell membranes against the concentration gradient (from areas of lower concentration to areas of higher concentration).

115
Q
The nurse observes edema in a patient who is experiencing venous congestion as a result of right heart failure. Which type of pressure facilitated the formation of the patient’s edema?
a.
Osmotic
b.
Oncotic
c.
Hydrostatic
d.
Concentration
A

ANS: C
Venous congestion increases capillary hydrostatic pressure. Increased hydrostatic pressure causes edema by causing increased movement of fluid into the interstitial area. Osmotic and oncotic pressures involve the concentrations of solutes and can contribute to edema in other situations, such as inflammation or malnutrition. Concentration pressure is not a nursing term.

116
Q
The nurse administers an intravenous (IV) hypertonic solution to a patient expects the fluid shift to occur in what direction?
a.
From intracellular to extracellular
b.
From extracellular to intracellular
c.
From intravascular to intracellular
d.
From intravascular to interstitial
A

ANS: A
Hypertonic solutions will move fluid from the intracellular to the extracellular (intravascular). A hypertonic solution has a concentration greater than normal body fluids, so water will shift out of cells because of the osmotic pull of the extra particles. Movement of water from the extracellular (intravascular) into cells (intracellular) occurs when hypotonic fluids are administered. Distribution of fluid between intravascular and interstitial spaces occurs by filtration, the net sum of hydrostatic and osmotic pressures.

117
Q
The nurse is laboratory blood results will expect to observe which cation in the most abundance?
a.
Sodium
b.
Chloride
c.
Potassium
d.
Magnesium
A

ANS: A
Sodium is the most abundant cation in the blood. Potassium is the predominant intracellular cation. Chloride is an anion (negatively charged) rather than a cation (positively charged). Magnesium is found predominantly inside cells and in bone.

118
Q
The nurse receives the patient’s most recent blood work results. Which laboratory value is of greatest concern?
a.
Sodium of 145 mEq/L
b.
Calcium of 15.5 mg/dL
c.
Potassium of 3.5 mEq/L
d.
Chloride of 100 mEq/L
A

ANS: B
Normal calcium range is 9 to 10.5 mg/dL; therefore, a value of 15.5 mg/dL is abnormally high and of concern. The rest of the laboratory values are within their normal ranges: sodium 136 to 145 mEq/L, potassium 3.5 to 5.0 mEq/L, and chloride 98 to 106 mEq/L.

119
Q
The nurse observes that the patient’s calcium is elevated. When checking the phosphate level, what does the nurse expect to see?
a.
An increase
b.
A decrease
c.
Equal to calcium
d.
No change in phosphate
A

ANS: B
Phosphate will decrease. Serum calcium and phosphate have an inverse relationship. When one is elevated, the other decreases, except in some patients with end-stage renal disease.

120
Q

Four patients arrive at the emergency department at the same time. Which patient will the nurse see first?
a.
An infant with temperature of 102.2° F and diarrhea for 3 days
b.
A teenager with a sprained ankle and excessive edema
c.
A middle-aged adult with abdominal pain who is moaning and holding her stomach
d.
An older adult with nausea and vomiting for 3 days with blood pressure 112/60

A

ANS: A
The infant should be seen first. An infant’s proportion of total body water (70% to 80% total body weight) is greater than that of children or adults. Infants and young children have greater water needs and immature kidneys. They are at greater risk for extracellular volume deficit and hypernatremia because body water loss is proportionately greater per kilogram of weight. A teenager with excessive edema from a sprained ankle can wait. A middle-aged adult moaning in pain can wait as can an older adult with a blood pressure of 112/60.

121
Q
A 2-year-old child has ingested a quantity of a medication that causes respiratory depression. For which acid-base imbalance will the nurse most closely monitor this child?
a.
Respiratory alkalosis
b.
Respiratory acidosis
c.
Metabolic acidosis
d.
Metabolic alkalosis
A

ANS: B
Respiratory depression leads to hypoventilation. Hypoventilation results in retention of CO2 and respiratory acidosis. Respiratory alkalosis would result from hyperventilation, causing a decrease in CO2 levels. Metabolic acid-base imbalance would be a result of kidney dysfunction, vomiting, diarrhea, or other conditions that affect metabolic acids.

122
Q
A patient is admitted for a bowel obstruction and has had a nasogastric tube set to low intermittent suction for the past 3 days. Which arterial blood gas values will the nurse expect to observe?
a.
Respiratory alkalosis
b.
Metabolic alkalosis
c.
Metabolic acidosis
d.
Respiratory acidosis
A

ANS: B
The patient is losing acid from the nasogastric tube so the patient will have metabolic alkalosis. Lung problems will produce respiratory alkalosis or acidosis. Metabolic acidosis will occur when too much acid is in the body like kidney failure.

123
Q
Which blood gas result will the nurse expect to observe in a patient with respiratory alkalosis?
a.
pH 7.60, PaCO2 40 mm Hg, HCO3– 30 mEq/L
b.
pH 7.53, PaCO2 30 mm Hg, HCO3– 24 mEq/L
c.
pH 7.35, PaCO2 35 mm Hg, HCO3– 26 mEq/L
d.
pH 7.25, PaCO2 48 mm Hg, HCO3– 23 mEq/L
A

ANS: B
Respiratory alkalosis should show an alkalotic pH and decreased CO2 (respiratory) values, with a normal HCO3–. In this case, pH 7.53 is alkaline (normal = 7.35 to 7.45), PaCO2 is 30 (normal 35 to 45 mm Hg), and HCO3– is 24 (normal = 22 to 26 mEq/L). A result of pH 7.60, PaCO2 40 mm Hg, HCO3– 30 mEq/L is metabolic alkalosis. pH 7.35, PaCO2 35 mm Hg, HCO3– 26 mEq/L is within normal limits. pH 7.25, PaCO2 48 mm Hg, HCO3– 23 mEq/L is respiratory acidosis.

124
Q
A nurse is caring for a patient whose electrocardiogram (ECG) presents with changes characteristic of hypokalemia. Which assessment finding will the nurse expect?
a.
Dry mucous membranes
b.
Abdominal distention
c.
Distended neck veins
d.
Flushed skin
A

ANS: B
Signs and symptoms of hypokalemia are muscle weakness, abdominal distention, decreased bowel sounds, and cardiac dysrhythmias. Distended neck veins occur in fluid overload. Thready peripheral pulses indicate hypovolemia. Dry mucous membranes and flushed skin are indicative of dehydration and hypernatremia.

125
Q

In which patient will the nurse expect to see a positive Chvostek’s sign?
a.
A 7-year-old child admitted for severe burns
b.
A 24-year-old adult admitted for chronic alcohol abuse
c.
A 50-year-old patient admitted for an acute exacerbation of hyperparathyroidism
d.
A 75-year-old patient admitted for a broken hip related to osteoporosis

A

ANS: B
A positive Chvostek’s sign is representative of hypocalcemia or hypomagnesemia. Hypomagnesemia is common with alcohol abuse. Hypocalcemia can be brought on by alcohol abuse and pancreatitis (which also can be affected by alcohol consumption). Burn patients frequently experience extracellular fluid volume deficit. Hyperparathyroidism causes hypercalcemia. Immobility is associated with hypercalcemia.

126
Q
A patient is experiencing respiratory acidosis. Which organ system is responsible for compensation in this patient?
a.
Renal
b.
Endocrine
c.
Respiratory
d.
Gastrointestinal
A
ANS:	A
The kidneys (renal) are responsible for respiratory acidosis compensation. A problem with the respiratory system causes respiratory acidosis, so another organ system (renal) needs to compensate. Problems with the gastrointestinal and endocrine systems can cause acid-base imbalances, but these systems cannot compensate for an existing imbalance.
127
Q
A nurse is caring for a patient prescribed peripheral intravenous (IV) therapy. Which task will the nurse assign to the nursing assistive personnel?
a.
Recording intake and output
b.
Regulating intravenous flow rate
c.
Starting peripheral intravenous therapy
d.
Changing a peripheral intravenous dressing
A

ANS: A
A nursing assistive personnel (AP) can record intake and output. An RN cannot delegate regulating flow rate, starting an IV, or changing an IV dressing to an NAP.

128
Q
The nurse is caring for a diabetic patient in renal failure who is in metabolic acidosis. Which laboratory findings are consistent with metabolic acidosis?
a.
pH 7.3, PaCO2 36 mm Hg, HCO3– 19 mEq/L
b.
pH 7.5, PaCO2 35 mm Hg, HCO3– 35 mEq/L
c.
pH 7.32, PaCO2 47 mm Hg, HCO3– 23 mEq/L
d.
pH 7.35, PaCO2 40 mm Hg, HCO3– 25 mEq/L
A

ANS: A
The laboratory values that reflect metabolic acidosis are pH 7.3, PaCO2 36 mm Hg, HCO3– 19 mEq/L. A laboratory finding of pH 7.5, PaCO2 35 mm Hg, HCO3– 35 mEq/L is metabolic alkalosis. pH 7.32, PaCO2 47 mm Hg, HCO3– 23 mEq/L is respiratory acidosis. pH 7.35, PaCO2 40 mm Hg, HCO3– 25 mEq/L values are within normal range.

129
Q
The nurse is assessing a patient and notes crackles in the lung bases and neck vein distention. Which action will the nurse take first?
a.
Offer calcium-rich foods.
b.
Administer diuretic.
c.
Raise head of bed.
d.
Increase fluids.
A

ANS: C
The patient is in fluid overload. Raising the head of the bed to a high-Flower’s position will help ease breathing and so is the first action. Offering calcium-rich foods is for hypocalcemia, not fluid overload. Administering a diuretic is the second action. Increasing fluids is contraindicated and would make the situation worse.

130
Q

A patient receiving chemotherapy has gained 5 pounds in 2 days. Which assessment question by the nurse is most appropriate?
a.
“Are you following any weight loss program?”
b.
“How many calories a day do you consume?”
c.
“Do you have dry mouth or feel thirsty?”
d.
“How many times a day do you urinate?”

A

ANS: D
A rapid gain in weight usually indicates extracellular volume (ECV) excess if the person began with normal ECV. Asking the patient about urination habits will help determine whether the body is trying to excrete the excess fluid or if renal dysfunction is contributing to ECV excess. This is too rapid a weight gain to be dietary; it is fluid retention. Asking about following a weight loss program will not help determine the cause of the problem. Caloric intake does not account for rapid weight changes. Dry mouth and thirst accompany ECV deficit, which would be associated with rapid weight loss.

131
Q
The health care provider has ordered a hypotonic intravenous (IV) solution to be administered. Which IV bag will the nurse prepare?
a.
0.45% sodium chloride (1/2 NS)
b.
0.9% sodium chloride (NS)
c.
Lactated Ringer’s (LR)
d.
Dextrose 5% in Lactated Ringer’s (D5LR)
A

ANS: A

0.45% sodium chloride is a hypotonic solution. NS and LR are isotonic. D5LR is hypertonic.

132
Q

The health care provider asks the nurse to monitor the fluid volume status of a heart failure patient and a patient at risk for clinical dehydration. Which is the most effective nursing intervention for monitoring both of these patients?
a.
Assess the patients for edema in extremities.
b.
Ask the patients to record their intake and output.
c.
Weigh the patients every morning before breakfast.
d.
Measure the patients’ blood pressures every 4 hours.

A

ANS: C
An effective measure of fluid retention or loss is daily weights; each kg (2.2 pounds) change is equivalent to 1 L of fluid gained or lost. This measurement should be performed at the same time every day using the same scale and the same amount of clothing. Although intake and output records are important assessment measures, some patients are not able to keep their own records themselves. Blood pressure can decrease with extracellular volume (ECV) deficit but will not necessarily increase with recent ECV excess (heart failure patient). Edema occurs with ECV excess but not with clinical dehydration.

133
Q

A nurse is caring for a patient diagnosed with cancer who presents with anorexia, blood pressure 100/60, and elevated white blood cell count. Which primary purpose for starting total parenteral nutrition (TPN) will the nurse add to the care plan?
a.
Stimulate the patient’s appetite to eat.
b.
Deliver antibiotics to fight off infection.
c.
Replace fluid, electrolytes, and nutrients.
d.
Provide medication to raise blood pressure.

A

ANS: C
Total parenteral nutrition is an intravenous solution composed of nutrients and electrolytes to replace the ones the patient is not eating or losing. TPN does not stimulate the appetite. TPN does not contain blood pressure medication or antibiotics.

134
Q
A patient presents to the emergency department with reports of vomiting and diarrhea for the past 48 hours. The health care provider orders isotonic intravenous (IV) therapy. Which IV will the nurse prepare?
a.
0.225% sodium chloride (1/4 NS)
b.
0.45% sodium chloride (1/2 NS)
c.
0.9% sodium chloride (NS)
d.
3% sodium chloride (3% NaCl)
A

ANS: C
Patients with prolonged vomiting and diarrhea become hypovolemic. A solution to replace extracellular volume is 0.9% sodium chloride, which is an isotonic solution. 0.225% and 0.45% sodium chloride are hypotonic. 3% sodium chloride is hypertonic.

135
Q
A nurse administering a diuretic to a patient is teaching about foods to increase in the diet. Which food choices by the patient will best indicate successful teaching?
a.
Milk and cheese
b.
Potatoes and fresh fruit
c.
Canned soups and vegetables
d.
Whole grains and dark green leafy vegetables
A

ANS: B
Potatoes and fruits are high in potassium. Milk and cheese are high in calcium. Canned soups and vegetables are high in sodium. Whole grains and dark green leafy vegetables are high in magnesium.

136
Q

The nurse is evaluating the effectiveness of the intravenous fluid therapy in a patient with hypernatremia. Which finding indicates goal achievement?
a.
Urine output increases to 150 mL/hr.
b.
Systolic and diastolic blood pressure decreases.
c.
Serum sodium concentration returns to normal.
d.
Large amounts of emesis and diarrhea decrease.

A

ANS: C
Hypernatremia is diagnosed by elevated serum sodium concentration. Blood pressure is not an accurate indicator of hypernatremia. Emesis and diarrhea will not stop because of intravenous therapy. Urine output is influenced by many factors, including extracellular fluid volume. A large dilute urine output can cause further hypernatremia.

137
Q
The nurse is calculating intake and output on a patient. The patient drinks 150 mL of orange juice at breakfast, voids 125 mL after breakfast, vomits 250 mL of greenish fluid, sucks on 60 mL of ice chips, and for lunch consumes 75 mL of chicken broth. Which totals for intake and output will the nurse document in the patient’s medical record?
a.
Intake 255; output 375
b.
Intake 285; output 375
c.
Intake 505; output 125
d.
Intake 535; output 125
A

ANS: A
Intake = 150 mL of orange juice, 60 mL of ice chips (but only counts as 30 since ice chips are half of the amount), and 75 mL of chicken broth; 150 + 30 + 75 = 255. Output = 125 mL of urine (void) and 250 mL of vomitus; 125 + 250 = 375.

138
Q
Which assessment finding should cause a nurse to further assess for extracellular fluid volume deficit?
a.
Moist mucous membranes
b.
Postural hypotension
c.
Supple skin turgor
d.
Pitting edema
A

ANS: B
Physical examination findings of deficit include postural hypotension, tachycardia, thready pulse, dry mucous membranes, and poor skin turgor. Pitting edema indicates that the patient may be retaining excess extracellular fluid.

139
Q
A patient is to receive 1000 mL of 0.9% sodium chloride intravenously at a rate of 125 mL/hr. The nurse is using microdrip gravity drip tubing. Which rate will the nurse calculate for the minute flow rate (drops/min)?
a.
12 drops/min
b.
24 drops/min
c.
125 drops/min
d.
150 drops/min
A

ANS: C
Microdrip tubing delivers 60 drops/mL. Calculation for a rate of 125 mL/hr using microdrip tubing: (125 mL/1 hr)(60 drops/1 mL)(1 hr/60 min) = 125 drop/min.

140
Q
A nurse begins infusing a 250-mL bag of IV fluid at 1845 on Monday and programs the pump to infuse at 50 mL/hr. At what time should the infusion be completed?
a.
2300 Monday
b.
2345 Monday
c.
0015 Tuesday
d.
0045 Tuesday
A

ANS: B
250 mL ÷ 50 mL/hr = 5 hr
1845 + 5 hr = 2345, which would be 2345 on Monday.

141
Q

A nurse caring for a diabetic patient with a bowel obstruction has orders to ensure that the volume of intake matches the output. In the past 4 hours, the patient received dextrose 5% with 0.9% sodium chloride through a 22-gauge catheter infusing at 150 mL/hr and has eaten 200 mL of ice chips. The patient also has an NG suction tube set to low continuous suction that had 300-mL output. The patient has voided 400 mL of urine. After reporting these values to the health care provider, which order does the nurse anticipate?
a.
Add a potassium supplement to replace loss from output.
b.
Decrease the rate of intravenous fluids to 100 mL/hr.
c.
Administer a diuretic to prevent fluid volume excess.
d.
Discontinue the nasogastric suctioning.

A

ANS: A
The total fluid intake and output equals 700 mL, which meets the provider goals. Patients with nasogastric suctioning are at risk for potassium deficit, so the nurse would anticipate a potassium supplement to correct this condition. Remember to record half the volume of ice chips when calculating intake. The other measures would be unnecessary because the net fluid volume is equal.

142
Q
A nurse is caring for a patient who is receiving peripheral intravenous (IV) therapy. When the nurse is flushing the patient’s peripheral IV, the patient reports pain. Upon assessment, the nurse notices a red streak that is warm to the touch. What is the nurse’s initial action?
a.
Record a phlebitis grade of 4.
b.
Assign an infiltration grade.
c.
Apply moist compress.
d.
Discontinue the IV.
A

ANS: D
The IV site has phlebitis. The nurse should discontinue the IV. The phlebitis score is 3. The site has phlebitis, not infiltration. A moist compress may be needed after the IV is discontinued.

143
Q

A nurse is assisting the health care provider in inserting a central line. Which action indicates the nurse is following the recommended bundle protocol to reduce central line-associated bloodstream infections (CLABSI)?
a.
Preps skin with povidone-iodine solution.
b.
Suggests the femoral vein for insertion site.
c.
Applies double gloving without hand hygiene.
d.
Uses chlorhexidine skin antisepsis prior to insertion.

A

ANS: D
A recommended bundle at insertion of a central line is hand hygiene prior to catheter insertion, use of maximum sterile barrier precautions upon insertion, chlorhexidine skin antisepsis prior to insertion and during dressing changes, avoidance of the femoral vein for central venous access for adults, and daily evaluation of line necessity, with prompt removal of nonessential lines. Povidone-iodine is not recommended.

144
Q

he nurse is caring for a group of patients. Which patient will the nurse see first?
a.
A patient with D5W hanging with the blood
b.
A patient with type A blood receiving type O blood
c.
A patient with intravenous potassium chloride that is diluted
d.
A patient with a right mastectomy and an intravenous site in the left arm

A

ANS: A
The nurse will see the patient with D5W and blood to prevent a medication error. When preparing to administer blood, prime the tubing with 0.9% sodium chloride (normal saline) to prevent hemolysis or breakdown of RBCs. All the rest are normal. A patient with type A blood can receive type O. Type O is considered the universal donor. A patient with a mastectomy should have the IV in the other arm. Potassium chloride should be diluted, and it is never given IV push.

145
Q
A nurse is administering a blood transfusion. Which assessment finding will the nurse report immediately?
a.
Blood pressure 110/60
b.
Temperature 101.3° F
c.
Poor skin turgor and pallor
d.
Heart rate of 100 beats/min
A

ANS: B
A fever should be reported immediately, and the blood transfusion stopped. All other assessment findings are expected. Blood is given to elevate blood pressure, improve pallor, and decrease tachycardia.

146
Q
A nurse has just received a bag of packed red blood cells (RBCs) for a patient. What is the longest time the nurse can let the blood infuse?
a.
30 minutes
b.
2 hours
c.
4 hours
d.
6 hours
A

ANS: C
Ideally a unit of whole blood or packed RBCs is transfused in 2 hours. This time can be lengthened to 4 hours if the patient is at risk for extracellular volume excess. Beyond 4 hours there is a risk for bacterial contamination of the blood.

147
Q

A patient has an acute intravascular hemolytic reaction to a blood transfusion. After discontinuing the blood transfusion, which is the nurse’s next action?
a.
Discontinue the IV catheter.
b.
Return the blood to the blood bank.
c.
Run normal saline through the existing tubing.
d.
Start normal saline at TKO rate using new tubing.

A

ANS: D
The nurse should first attach new tubing and begin running in normal saline at a rate to keep the vein open, in case any medications need to be delivered through an IV site. The existing tubing should not be used because that would infuse the blood in the tubing into the patient. It is necessary to preserve the IV catheter in place for IV access to treat the patient. After the patient has been assessed and stabilized, the blood can be returned to the blood bank.

148
Q
A nurse assessing a patient who is receiving a blood transfusion finds that the patient is anxiously fidgeting in bed. The patient is afebrile but dyspneic. The nurse auscultates crackles in both lung bases and sees jugular vein distention. On which transfusion complication will the nurse focus interventions?
a.
Fluid volume excess
b.
Hemolytic reaction
c.
Anaphylactic shock
d.
Septicemia
A

A
The signs and symptoms are concurrent with fluid volume excess. Anaphylactic shock would have presented with urticaria, dyspnea, and hypotension. Septicemia would include a fever. A hemolytic reaction would consist of flank pain, chills, and fever.

149
Q

A nurse preparing to start a blood transfusion will use which type of tubing?
a.
Two-way valves to allow the patient’s blood to mix and warm the blood transfusing
b.
An injection port to mix additional electrolytes into the blood
c.
One with a filter to ensure that clots do not enter the patient
d.
An air vent to let bubbles into the blood

A

ANS: C
When administering a transfusion, you need an appropriate-size IV catheter and blood administration tubing that has a special in-line filter. The patient’s blood should not be mixed with the infusion blood. Air bubbles should not be allowed to enter the blood. The only substance compatible with blood is normal saline; no additives should be mixed with the infusing blood.

150
Q
The nurse is caring for a patient with hyperkalemia. Which body system assessment is the priority?
a.
Gastrointestinal
b.
Neurological
c.
Respiratory
d.
Cardiac
A

ANS: D
Cardiac is the priority. Hyperkalemia places the patient at risk for potentially serious dysrhythmias and cardiac arrest. Potassium balance is necessary for cardiac function. Respiratory is the priority with hypokalemia. Monitoring of gastrointestinal and neurological systems would be indicated for other electrolyte imbalances.

151
Q
Which assessment finding will the nurse expect for a patient with the following laboratory values: sodium 145 mEq/L, potassium 4.5 mEq/L, calcium 4.5 mg/dL?
a.
Weak quadriceps muscles
b.
Decreased deep tendon reflexes
c.
Light-headedness when standing up
d.
Tingling of extremities with possible tetany
A

ANS: D
This patient has hypocalcemia because the normal calcium range is 8.4 to 10.5 mg/dL. Hypocalcemia causes muscle tetany, positive Chvostek’s sign, and tingling of the extremities. Sodium and potassium values are within their normal ranges: sodium 135 to 145 mEq/L; potassium 3.5 to 5.0 mEq/L. Light-headedness when standing up is a manifestation of ECV deficit or sometimes hypokalemia. Weak quadriceps muscles are associated with potassium imbalances. Decreased deep tendon reflexes are related to hypercalcemia or hypermagnesemia.

152
Q

While the nurse is taking a patient history, the nurse discovers the patient has a type of diabetes that results from a head injury and does not require insulin. Which dietary change should the nurse share with the patient?
a.
Reduce the quantity of carbohydrates ingested to lower blood sugar.
b.
Include a serving of dairy in each meal to elevate calcium levels.
c.
Drink plenty of fluids throughout the day to stay hydrated.
d.
Avoid foods high in acid to avoid metabolic acidosis.

A

ANS: C
The patient has diabetes insipidus, which places the patient at risk for dehydration and hypernatremia. Dehydration should be prevented by drinking plenty of fluids to replace the extra water excreted in the urine. Foods high in acid are not what causes metabolic acidosis. A reduction in carbohydrates to lower blood sugar will not help a patient with diabetes insipidus but it may help a patient with diabetes mellitus. Calcium-rich dairy products would be recommended for a hypocalcemic patient.

153
Q
A nurse is selecting a site to insert an intravenous (IV) catheter on an adult. Which actions will the nurse take? (Select all that apply.)
a.
Check for contraindications to the extremity.
b.
Start proximally and move distally on the arm.
c.
Choose a vein with minimal curvature.
d.
Choose the patient’s dominant arm.
e.
Select a vein that is rigid.
f.
Avoid areas of flexion.
A

ANS: A, C, F
The vein should be relatively straight to avoid catheter occlusion. Contraindications to starting an IV catheter are conditions such as mastectomy, AV fistula, and central line in the extremity and should be checked before initiation of IV. Avoid areas of flexion if possible. The nurse should start distally and move proximally, choosing the nondominant arm if possible. The nurse should feel for the best location; a good vein should feel spongy; a rigid vein should be avoided because it might have had previous trauma or damage.

154
Q
Which assessments will alert the nurse that a patient’s IV has infiltrated? (Select all that apply.)
a.
Edema of the extremity near the insertion site
b.
Reddish streak proximal to the insertion site
c.
Skin discolored or pale in appearance
d.
Pain and warmth at the insertion site
e.
Palpable venous cord
f.
Skin cool to the touch
A

ANS: A, C, F
Infiltration results in skin that is edematous near the IV insertion site. Skin is cool to the touch and may be pale or discolored. Pain, warmth, erythema, a reddish streak, and a palpable venous cord are all symptoms of phlebitis.

155
Q

A nurse is discontinuing a patient’s peripheral IV access. Which actions should the nurse take? (Select all that apply.)
a.
Wear sterile gloves and a mask.
b.
Stop the infusion before removing the IV catheter.
c.
Use scissors to remove the IV site dressing and tape.
d.
Apply firm pressure with sterile gauze during removal.
e.
Keep the catheter parallel to the skin while removing it.
f.
Apply pressure to the site for 2 to 3 minutes after removal

A

ANS: B, E, F
The nurse should stop the infusion before removing the IV catheter, so the fluid does not drip on the patient’s skin; keep the catheter parallel to the skin while removing it to reduce trauma to the vein; and apply pressure to the site for 2 to 3 minutes after removal to decrease bleeding from the site. Scissors should not be used because they may accidentally cut the catheter or tubing or may injure the patient. During removal of the IV catheter, light pressure, not firm pressure, is indicated to prevent trauma. Clean gloves are used for discontinuing a peripheral IV access because gloved hands will handle the external dressing, tubing, and tape, which are not sterile.

156
Q

A nursing student asks the nurse to explain the role of cyclooxygenase-2 (COX-2) and its role in inflammation. The nurse will explain that COX-2:

a. converts arachidonic acid into a chemical mediator for inflammation.
b. directly causes vasodilation and increased capillary permeability.
c. irritates the gastric mucosa to cause gastrointestinal upset.
d. releases prostaglandins, which cause inflammation and pain in tissues.

A

ANS: A
COX-2 is an enzyme that converts arachidonic acid into prostaglandins and their products, and this synthesis causes pain and inflammation. They do not act directly to cause inflammation. COX-1 irritates the gastric mucosa. COX-2 synthesizes but does not release prostaglandins.

157
Q

A nursing student asks how nonsteroidal anti-inflammatory drugs (NSAIDs) work to suppress
inflammation and reduce pain. The nurse will explain that NSAIDs:
a. exert direct actions to cause relaxation of smooth muscle.
b. inhibit the enzyme cyclooxyrgenase that is necessary for prostaglandin synthesis
c. interfere with neuronal pathways associated with prostaglandin action
d. suppress prostaglandin activity by blocking tissue receptor sites

A

ANS: B
NSAIDs act by inhibiting COX-1 and COX-2 to help block prostaglandin synthesis. They do not have direct action on tissues, nor do they interfere with chemical receptor sites or neuronal pathways.

158
Q

A patient is taking ibuprofen 400 mg every 4 hours to treat moderate arthritis pain and reports that it is less effective than before. What action will the nurse take?
a. Counsel the patient to discuss a change in dose or changing to a prescription
NSAID with the provider.
b. Recommend adding aspirin to increase the antiinflammatory effect.
c. Suggest asking the provider about a short course of corticosteroids.
d. Tell the patient to increase the dose to 800 mg every 4 hours.

A

ANS: A
The patient should discuss a possible change in dose or changing to a prescription NSAID with the provider if tolerance has developed to the over-the-counter NSAID. Patients should not take aspirin with NSAIDs because of the increased risk of bleeding and gastrointestinal upset. Steroids are not the drugs of choice for arthritis because of their side effects and are not used unless inflammation is severe. A prescription NSAID would generally be used prior to starting corticosteroids. Increasing the dose will increase the potential for side effects but may not increase desired effects. It is important that the patient discuss any changes with their provider and not self-titrate the NSAID.

159
Q

A patient who is taking aspirin for arthritis pain asks the nurse why it also causes gastrointestinal upset. The nurse understands that this is because aspirin:

a. increases gastrointestinal secretions.
b. increases hypersensitivity reactions.
c. inhibits both COX-1 and COX-2.
d. selectively inhibits COX-2.

A

ANS: C
Aspirin is a non-specific COX-1 and COX-2 inhibitor. COX-1 protects the stomach lining, so when it is inhibited, gastric upset can occur. Aspirin does not increase gastrointestinal secretions or hypersensitivity reactions.

160
Q

A patient is taking aspirin for secondary prevention of myocardial infarction and is experiencing moderate gastrointestinal upset. The nurse will contact the patient’s provider to discuss changing from aspirin to which of the following?

a. A COX-2 inhibitor
b. Celecoxib (Celebrex)
c. Enteric-coated aspirin
d. Ibuprofen

A

ANS: C
Aspirin is often used to inhibit platelet aggregation for cardiovascular prevention. Patients taking aspirin for this purpose would not benefit from COX-2 inhibitors, since the COX-1 enzyme is responsible for inhibiting platelet aggregation. The patient may benefit from taking an enteric-coated aspirin product to lessen the gastrointestinal distress. Celecoxib and is a COX-2 selective inhibitor. Ibuprofen is not indicated for cardiovascular event prevention.

161
Q

The nurse is performing a health history on a patient who has arthritis. The patient reports tinnitus. Suspecting a drug adverse effect, the nurse will ask the patient about which medication?

a. Aspirin (Bayer)
b. Acetaminophen (Tylenol)
c. Anakinra (Kineret)
d. Prednisone (Deltasone)

A

ANS: A
Aspirin causes tinnitus at low toxicity levels or in patients with hypersensitivity to aspirin. The nurse should question the patient about this medication. The other medications are less likely to contribute to this side effect.

162
Q

The nurse is teaching a patient about using high-dose aspirin to treat arthritis. What information will the nurse include when teaching this patient?

a. “A normal serum aspirin level is between 30 and 40 mg/dL.”
b. “You may need to stop taking this drug a week prior to surgery.”
c. “You will need to monitor aspirin levels if you are also taking warfarin.”
d. “Your stools may become dark, but this is a harmless side effect.”

A

ANS: B
Aspirin should be discontinued prior to surgery to avoid prolonged bleeding time. A normal serum level is 15 to 30 mg/dL. Patients taking warfarin and aspirin will have increased amounts of warfarin, so the INR will need to be monitored. Tarry stools are a symptom of gastrointestinal bleeding and should be reported.

163
Q

A patient who takes high-dose aspirin to treat rheumatoid arthritis has a serum salicylate level of 35 mg/dL. The nurse will perform which action?

a. Assess the patient for signs of toxicity, such as tinnitus.
b. Monitor the patient for signs of Reye’s syndrome.
c. Notify the provider of severe aspirin toxicity.
d. Request an order for an increased aspirin dose.

A

ANS: A
Mild toxicity occurs at levels above 30 mg/dL, so the nurse should assess for signs of toxicity, such as tinnitus. This level will not increase the risk for Reye’s syndrome. Severe toxicity occurs at levels greater than 50 mg/dL. The dose should not be increased.

164
Q

The nurse provides teaching for a patient who will begin taking indomethacin (Inderal) to treat rheumatoid arthritis. Which statement by the patient indicates a need for further teaching?

a. “I should limit sodium intake while taking this drug.”
b. “I should take indomethacin on an empty stomach.”
c. “I will need to check my blood pressure frequently.”
d. “I will take the medication twice daily.”

A

ANS: B
Indomethacin is very irritating to the stomach and should be taken with food. It can cause sodium retention and elevated blood pressure, so patients should limit sodium intake. The medication is taken twice daily.

165
Q

The nurse is caring for a postpartum woman who is refusing opioid analgesics but is rating her pain as a 7 or 8 on a 10-point pain scale. The nurse will contact the provider to request an order for which analgesic medication?

a. Diclofenac sodium (Voltaren)
b. Ketoprofen (Orudis)
c. Ketorolac (Toradol)
d. Naproxen (Naprosyn)

A

ANS: C
Ketorolac is an injectable NSAID and has shown analgesic efficacy equal or superior to that of opioid analgesics. The other NSAIDs listed are not used for postoperative pain.

166
Q

A patient who has osteoarthritis with mild to moderate pain asks the nurse about taking
over-the-counter ibuprofen (Motrin). What will the nurse tell this patient?
a. “It may take several weeks to achieve maximum therapeutic effects.”
b. “Unlike aspirin, there is no increased risk of bleeding with ibuprofen.”
c. “Take ibuprofen twice daily for maximum analgesic benefit.”
d. “Combine ibuprofen with naproxen for best effect.”

A

ANS: A
OTC NSAIDs can be effective for mild to moderate arthritis pain, but the full effects may not appear for several weeks. NSAIDs carry a risk for bleeding. Ibuprofen is taken TID or QID. Ibuprofen and naproxen are both NSAIDs and ideally should not be combined.

167
Q

The nurse is caring for a patient who has been taking an NSAID for 4 weeks for osteoarthritis. The patient reports decreased pain, but the nurse notes continued swelling of the affected joints. The nurse will perform which action?

a. Assess the patient for drug-seeking behaviors.
b. Notify the provider that the drug is not effective.
c. Reassure the patient that swelling will decrease eventually.
d. Remind the patient that this drug is given for pain only.

A

ANS: B
This medication is effective for both pain and swelling. After 4 weeks, there should be some decrease in swelling, so the nurse should report that this medication is ineffective. There is no indication that this patient is seeking an opioid analgesic. The drug should be effective within several weeks. NSAIDs are given for pain and swelling.

168
Q

The nurse is caring for a patient who has rheumatoid arthritis and who is receiving infliximab (Remicade) IV every 8 weeks. Which laboratory test will the nurse anticipate that this patient will need?

a. Calcium level
b. Complete blood count
c. Electrolytes
d. Potassium

A

ANS: B

Infliximab is an immunomodulator and can cause agranulocytosis, so patients should have regular CBC monitoring.

169
Q

The nurse is teaching a patient about taking colchicine to treat gout. What information will the nurse include when teaching this patient about this drug?

a. Avoid all alcohol except beer.
b. Include salmon in the diet.
c. Increase fluid intake
d. Take on an empty stomach

A

ANS: C
The patient who is taking colchicine should increase fluid intake to promote uric acid excretion and prevent renal calculi. Foods rich in purine should be avoided, including beer, and some sea foods, such as salmon. Gastric irritation is a common problem, so colchicine should be taken with food.

170
Q

Which antigout medication is considered first-line to treat chronic tophaceous gout?

a. Allopurinol (Zyloprim)
b. Colchicine
c. Probenecid (Benemid)
d. Celecoxib (Celebrex)

A

ANS: A
Allopurinol inhibits the biosynthesis of uric acid and is used long-term to manage chronic gout. Colchicine does not inhibit uric acid synthesis or promote uric acid secretion and is not used for chronic gout. Probenecid can be used for chronic gout but is not the first choice. Celecoxib is a COX-2 selective anti-inflammatory and does not have a role in treating chronic tophaceous gout.

171
Q

The nurse is assessing a patient who has gout who will begin taking allopurinol (Zyloprim). The nurse reviews the patient’s medical record and will be concerned about which of the following findings?

a. History of kidney stones
b. Increased serum uric acid
c. Slight increase in the white blood count
d. Increased serum glucose

A

ANS: A
Allopurinol use can increase the risk of kidney stones resulting from uric acid secretion. This can be prevented by increasing water intake and maintaining a urine pH above 6. A history of kidney stones would not be a contraindication to allopurinol use, but additional caution and patient teaching to prevent kidney stone formation is warranted.

172
Q

The nurse provides teaching for a patient who will begin taking allopurinol. Which statement by the patient indicates understanding of the teaching?

a. “I should increase my vitamin C intake.”
b. “I will get yearly eye exams.”
c. “I will increase my proteinNintake.”
d. “I will limit fluids to prevent edema.”

A

ANS: B
Patients taking allopurinol can have visual changes with prolonged use and should have yearly eye exams. It is not necessary to increase vitamin C. Protein can increase purine intake, which is not recommended. Patients should consume extra fluids.

173
Q

Which of the following DMARD medications is an IL-1 receptor antagonist?

a. Etanercept (Enbrel)
b. Anakinra (Kineret)
c. Infliximab (Remicade)
d. Rituximab (Rituxan)

A

ANS: B
Anakinra is a DMARD that works via antagonism of IL-1 receptors. Etanercept and infliximab are TNF inhibitors. Rituximab induces cell lysis of B cells. All of these agents are indicated to treat rheumatoid arthritis.

174
Q

Which are characteristic signs of inflammation? (Select all that apply.) a. Edema

b. Erythema
c. Heat
d. Numbness
e. Pallor
f. Paresthesia

A

ANS: A, B, C

Edema, erythema, and heat are signs of inflammation. The other three are signs of neurocirculatory compromise.

175
Q

A nurse is teaching the client with systemic lupus erythematosus about prednisone. What information is the priority?

a. Might make the client feel jittery or nervous.
b. Can cause sodium and fluid retention.
c. Long-term effects include fat redistribution.
d. Never stop prednisone abruptly

A

ANS: D
The nurse teaches the client to avoid stopping the drug abruptly as the priority because this can lead to a life-threatening adrenal crisis. Short-term side effects do include jitteriness or nervousness, sodium and water retention. One long-term side effect is fat redistribution resulting in “moon face” and “buffalo hump.”

176
Q

The nurse is caring for clients on the medical-surgical unit. What action by the nurse will help
prevent a client from having a type II hypersensitivity reaction?
a. Administering steroids for a positive TB test
b. Correctly identifying the client prior to a blood transfusion
c. Keeping the client free of the offending agent
d. Providing a latex-free environment for the client

A

ANS: B
A classic example of a type II hypersensitivity reaction is a blood transfusion reaction. The can be prevented by correctly identifying the client and cross-checking the unit of blood to be administered. A positive type IV response is a positive TB test. Avoidance therapy is the cornerstone of treatment for a type IV hypersensitivity to substances that are known and can be avoided such as poison ivy and insect stings. Latex allergies are a type I hypersensitivity

177
Q

A client has been newly diagnosed with systemic lupus erythematosus and is reviewing self-care measures with the nurse. Which statement by the client indicates a need to review the material?

a. “I will avoid direct sunlight as much as possible.”
b. “Baby powder is good for the constant sweating.”
c. “Grouping errands will help prevent fatigue.”
d. “Rest time will have to become a priority”

A

ANS: B
Constant sweating is not a sign of SLE and powders are drying so they should not be used, least not in excess. The client is correct in stating he/she should avoid direct sunlight, that grouping errands can prevent or reduce fatigue, and that rest will have to become a priority

178
Q

A client calls the clinic to report exposure to poison ivy and an itchy rash that is not helped
with over-the-counter antihistamines. What response by the nurse is most appropriate?
a. “Antihistamines do not help poison ivy.”
b. “There are different antihistamines to try.”
c. “You should be seen in the clinic right away.”
d. “You will need to take some IV steroids.”

A

ANS: A
Since histamine is not the mediator of a type IV reaction such as with poison ivy, antihistamines will not provide relief. The nurse would educate the client about this. The client does not need to be seen right away. The client may or may not need steroids; they may be given either IV or orally.

179
Q

A nurse has educated a client on an epinephrine autoinjector. What statement by the client
indicates additional instruction is needed?
a. “I don’t need to go to the hospital after using it.”
b. “I must carry two autoinjectors with me at all times.”
c. “I will write the expiration date on my calendar.”
d. “This can be injected right through my clothes”

A

ANS: A
Clients would be instructed to call 911 and go to the hospital for monitoring after using the autoinjector. The medication may wear off before the offending agent has cleared the client’s system. The other statements show good understanding of this treatment.

180
Q

A nurse has presented an educational program to a community group on Lyme disease. What
statement by a participant indicates the need to review the material?
a. “I should take precautions against ticks, especially in the summer.”
b. “A red rash that looks like a bull’s-eye may be one of the symptoms.”
c. “If Lyme disease is not treated successfully, it is usually fatal.”
d. “For Stage I disease, antibiotics are usually needed for 14 to 21 days.”

A

ANS: C
Untreated Lyme disease can lead to chronic complications, or Stage III Lyme disease, such as arthritis, chronic fatigue, memory/thinking problems. It is not usually a fatal disease so this information would need to be corrected by the nurse. The other participant statements are
correct.

181
Q

The nurse is studying hypersensitivity reactions. Which reactions are correctly matched with their hypersensitivity types? (Select all that apply.)

a. Type I—examples include hay fever and anaphylaxis.
b. Type II—mediated by action of immunoglobulin M (IgM).
c. Type III—immune complex deposits in blood vessel walls.
d. Type IV—examples are poison ivy and transplant rejection.
e. Type IV—involve both antibodies and complement.

A

ANS: A,C,D
Type I reactions are mediated by immunoglobulin E (IgE) and include hay fever, anaphylaxis,
and allergic asthma. Type III reactions consist of immune complexes that form and deposit in
the walls of blood vessels. Type IV reactions include responses to poison ivy exposure,
positive tuberculosis tests, and graft rejection. Type II reactions are mediated by immunoglobulin G, not IgM. Type IV hypersensitivity reactions do not involve either
antibodies or complement.

182
Q

The nurse is assessing a client for signs and symptoms of systemic lupus erythematosus
(SLE). Which of the following would be consistent with this disorder? (Select all that apply.)
a. Discoid rash on skin exposed to sunlight
b. Urinalysis positive for casts and protein c. Painful, deformed small joints
d. Pain on inspiration
e. Thrombocytosis
f. Serum positive for antinuclear antibodies (ANA)

A

ANS: A,B,D,F
Signs and symptoms of SLE include (but are not limited to) a discoid rash on skin exposed to the sun, urinalysis with casts and protein, pleurisy as manifested by pain on inspiration, and
positive ANA titers in the blood. Nonerosive arthritis in peripheral joints can occur but does
not lead to deformity. Thrombocytopenia is another sign.

183
Q

A client is being administered the first dose of belimumab for a systemic lupus erythematosus flare. What actions by the nurse are most appropriate? (Select all that apply.)

a. Observe the client for at least 2 hours afterwards
b. Instruct the client about the monthly infusion schedule.
c. Inform the client not to drive or sign legal papers for 24 hours.
d. Ensure emergency equipment is working and nearby.
abirb. com/test
e. Make a follow-up appointment for a lipid panel in 2 months. f. Instruct the client to hold other medications for 72 hours.

A

ANS: A,D
This drug is a monoclonal antibody to tumor necrosis factor. The first dose would be administered in a place where severe allergic reactions and/or anaphylaxis can be managed. This includes having emergency equipment nearby. The client would be observed for at least
2 hours after this first dose. This drug does not cause drowsiness, so there would be no restrictions on driving or signing legal documents. Elevated lipids are not associated with this drug. This drug is used in combination with other therapies, especially during a flare.

184
Q

A nurse caring for clients with systemic lupus erythematosus (SLE) plans care understanding the most common causes of death for these clients is which of the following? (Select all that apply.)

a. Infection
b. Cardiovascular impairment
c. Vasculitis
d. Chronic kidney disease
e. Liver failure
f. Blood dyscrasias

A

ANS: B,D
Any and all organs and tissues may be affected in SLE but the most common causes of death in clients with SLE include cardiovascular impairment and chronic kidney disease.

185
Q

The nurse learns that the most important function of inflammation and immunity is which purpose?

a. Destroying bacteria before damage occurs
b. Preventing any entry of foreign material
c. Providing maximum protection against infection
d. Regulating the process of self-tolerance

A

ANS: C
Immunity and Inflammation working together are critical to maintaining health, preventing disease, and repairing tissue damage. When all the different parts and functions of immunity are working well, the adult is immunocompetent and has maximum protection against infection. Working together, their function is not limited to destroying bacteria before damage occurs. They do not prevent the entry of all foreign materials and immunity alone regulates the process of self-tolerance

186
Q

A nurse is assessing an older client for the presence of infection. The client’s temperature is

  1. 6° F (36.4° C). What response by the nurse is best?
    a. Assess the client for more specific signs.
    b. Conclude that an infection is not present.
    c. Document findings and continue to monitor.
    d. Request the primary health care provider order blood cultures.
A

ANS: A
Because older adults have decreased immune function, including reduced neutrophil function, fever may not be present during an episode of infection. The nurse would assess the client for specific signs of infection. Documentation needs to occur, but a more thorough assessment comes first. Blood cultures may or may not be needed depending on the results of further assessment.

187
Q

A clinic nurse is working with an older client. What action is most important for preventing infections in this client?

a. Assessing vaccination records for booster shot needs
b. Encouraging the client to eat a nutritious diet
c. Instructing the client to wash minor wounds carefully
d. Teaching hand hygiene to prevent the spread of microbes

A

ANS: A
Older adults may have insufficient antibodies that have already been produced against microbes to which they have been exposed. Therefore, older adults need booster shots for many vaccinations they received as younger people. A nutritious diet, proper wound care, and hand hygiene are relevant for all populations.

188
Q

A client has a leg wound that is in Stage II of the inflammatory response. For what sign or symptom does the nurse assess?

a. Noticeable rubor
b. Purulent drainage
c. Swelling and pain
d. Warmth at the site

A

ANS: B
During the second phase of the inflammatory response, neutrophilia occurs, producing pus. Rubor (redness), swelling, pain, and warmth are cardinal signs of the general inflammatory process.

189
Q

A nurse learning about antibody-mediated immunity learns that the cell with the most direct role in this process begins development in which tissue or organ?

a. Bone marrow
b. Spleen
c. Thymus
d. Tonsils

A

ANS: A
The B-cell is the primary cell in antibody-mediated immunity and is released from the bone marrow. These cells then travaelbtoiortbhe.rcoorgamns /atned tsistsues, known as the secondary lymphoid tissues for B-cells.

190
Q

The nurse understands that which type of immunity is the longest acting?

a. Artificial active
b. Inflammatory
c. Natural active
d. Natural passive

A

ANS: C
Natural active immunity is the most effective and longest acting type of immunity. Artificial and natural passive do not last as long. “Inflammatory” is not a type of immunity.

191
Q

The nurse working with clients who have autoimmune diseases understands that what component of cell-mediated immunity is the problem?

a. CD4+ cells
b. Cytotoxic T-cells
c. Natural killer cells
d. Regulator T-cells

A

ANS: D
Regulator T-cells help prevent hypersensitivity to one’s own cells, which is the basis for
autoimmune disease. CD4+ cells are also known as helper/inducer cells, which secrete cytokines. Natural killer cells have direct cytotoxic effects on some non-self cells without first
being sensitized. Regulator T-cells have an inhibitory action on the immune system. Cytotoxic T-cells are effective against self cells infected by parasites such as viruses or protozoa.

192
Q

A primary health care provider notifies the nurse that a client has a “bandemia.” What action does the nurse anticipate?

a. Administer antibiotics
b. Place the client in isolation.
c. Administer IV leukocytes.
d. Obtain an immunization history

A

ANS: A
A bandemia, or shift to the left, in the white count differential means that an acute, continuing
infection has placed so much stress on the immune system that the most numerous type of
neutrophil in circulation are immature, or band cells. The nurse would anticipate administering antibiotics. The client may or may not need isolation. Leukocyte infusion and immunization history are not relevant.

193
Q

What does the nurse learn about the function of colony-stimulating factor?

a. Triggers the bone marrow to shorten the time needed to produce mature WBCs.
b. Causes capillary leak in acute inflammation.
c. Responsible for creating exudate (pus) at infectious sites.
d. Dilates blood vessels at the site of inflammation leading to hyperemia.

A

ANS: A
Colony-stimulating factor triggers the bone marrow to shorten the time needed to produce
mature WBCs from about 14 days to hours. Increased blood flow to the local area of
inflammation produces hyperemia, or redness. Exudate is formed by neutrophils and consists
of dead WBCs, necrotic tissue, and fluids that escape from damaged cells. Histamine, serotonin, and kinins dilate arterioles leading to redness and warmth.

194
Q

The older client’s adult child questions the nurse as to why the client is at higher risk for
infection when the client’s white cell count is within the normal range. What response by the
nurse is best?
a. “The white cell count does not tell us everything about immunity.”
b. “White blood cells are less active in older people so they are not as efficient.”
c. “Older people typically have poor nutrition which makes them prone to infection.”
d. “As one ages, immunoglobulins cease to be produced in response to illness.”

A

ANS: B
An age-related change in immunity is that neutrophils in the older adult are less active and
therefore less effective in immunity. The white blood cell count is not the only thing that can
inform about immunity, but this response is too vague to be useful. Many older adults do have
poor nutrition that does affect immunity, but this is not true for everyone and the stem does not contain information stating that is problematic for this older adult. Immunoglobulins do not cease to be produced with age.

195
Q

For a person to be immunocompetent, which processes need to be functional and interact appropriately with each other? (Select all that apply.)

a. Antibody-mediated immunity
b. Cell-mediated immunity abirb.com/test
c. Inflammation
d. Red blood cells
e. White blood cells

A

ANS: A,B,C
The three processes that need to be functional and interact with each other for a person to be immunocompetent are antibody-mediated immunity, cell-mediated immunity, and inflammation. Red and white blood cells are not processes.

196
Q

A nurse is learning about the types of different cells involved in the inflammatory response. Which principles does the nurse learn? (Select all that apply.)

a. Basophils are only involved in the general inflammatory process.
b. Eosinophils increase during allergic reactions and parasitic invasion.
c. Macrophages can participate in many episodes of phagocytosis.
d. Monocytes turn into macraopbhaigrebs.acfteor tmhey/teentesrtbody tissues.
e. Neutrophils can only take part in one episode of phagocytosis.

A

ANS: B,C,D,E
Eosinophils do increase durinag balliergbic.canod pmara/stieticsintvasion. Macrophages participate in
many episodes of phagocytosis. Monocytes turn into macrophages after they enter body tissues. Neutrophils only take part in one episode of phagocytosis. Basophils are involved in
both the general inflammatory response and allergic or hypersensitivity responses.

197
Q

The nurse assesses clients for the cardinal signs of inflammation. Which signs/symptoms does this include? (Select all that apply.)

a. Edema
b. Pulselessness
c. Pallor
d. Redness
e. Warmth
f. Decreased function

A

ANS: A,D,E,F
The five cardinal signs of inflammation include redness, warmth, pain, swelling, and
decreased function.

198
Q

Which are steps in the process of making an antigen-specific antibody? (Select all that apply.)

a. Antibody-antigen binding
b. Invasion
c. Opsonization
d. Recognition
e. Sensitization
f. Production

A

ANS: A,B,D,E,F
The seven steps in the process of making antigen-specific antibodies are: exposure/invasion,
antigen recognition, sensitization, antibody production and release, antigen-antibody binding,
antibody binding actions, and sustained immunity. Opsonization is the adherence of an antibody to the antigen, marking it for destruction.

199
Q

The nurse is learning about imambunirobglo.bcuolinms. ./Wtehischtprinciples does the nurse learn? (Select all that apply.)

a. IgA is found in high concentrations in secretions from mucous membranes.
b. IgD is present in the highest concentrations in mucous membranes.
c. IgE is associated with antibody-mediated hypersensitivity reactions.
d. IgG comprises the majority of the circulating antibody population.
e. IgM is the first antibody formed by a newly sensitized B-cell.

A
ANS: A,C,D,E
Immunoglobulin A (IgA) is found in high concentrations in secretions from mucous membranes. Immunoglobulin E (IgE) is associated with antibody-mediated hypersensitivity reactions. The majority of the circulating antibody population consists of immunoglobulin G (IgG). The first antibody formed by a newly sensitized B-cell is immunoglobulin M (IgM). Immunoglobulin D (IgD) is typically present in low concentrations.
200
Q

The nurse learns that which risk factors can affect immunity? (Select all that apply.)

a. Age
b. Environmental factors
c. Ethnicity
d. Drugs
e. Nutritional status

A

ANS: A,B,D,E
Immunity changes during an adult’s life as a result of nutritional status, environmental
conditions, drugs, disease, and age. Immunity is most efficient in young adults and older
adults have decreased immune function. Ethnicity does not affect immunity.

201
Q

The nurse is teaching an elderly client the risks of infection for older adults. Which of the
following factors would the nurse include in the education? (Select all that apply.)
a. Higher risk for respiratory tract and genitourinary infections.
b. May not have a fever with severe infection.
c. Show expected changes in white blood cell counts.
d. Should receive influenza, pneumococcal, and shingles vaccinations.
e. Skin tests for tuberculosis may be falsely negative.
f. Booster vaccinations are not likely needed as one ages.

A

ANS: A,B,D,E
Immunity changes during an adult’s life and older adults have decreased immune function. The number and function of neutrophils and macrophages are reduced leading to reduced response to infection and injury, such as temperature elevation. The usual response of an
increased white blood cell count is delayed or absent. Older adults are less able to make new antibodies in response to the presence of new antigens requiring repeat vaccinations and immunizations. Skin tests for tuberculosis may be falsely negative and there is an increased risk for bacterial and fungal infections due to the decreased number of circulating T-lymphocytes.

202
Q

A nurse is studying the functions of specific leukocytes. Which leukocytes are matched correctly with their functions? (Select all that apply)
a. Monocyte: matures into macrophage
b. Basophil: releases vasoactive amines during an allergic reaction.
abirb.com/test
c. Plasma cell: secretes immunoglobulins in response to the presence of a specific
antigen.
d. Cytotoxic T-cells: attacks and destroys ingested poisons and toxins.
e. Natural killer cell: nonselectively attacks non-self cells.
f. Regulator T-cells: become sensitized for self-recognition in the bone marrow.

A

ANS: A,C,E
Monocytes mature into macrophages, plasma cells secrete immunoglobulin in the presence of
specific antigens, and natural killer cells nonselectively attack non-self cells. Basophils release
histamines, kinins, and heparin in areas of tissue damage. Cytotoxic T-cells selectively attack
and destroy non-self cells, including virally infected cells, grafts, and transplanted organs. Regulator T-cells become sensitized for self-recognition in the thymus.

203
Q

A nurse is studying the function of immunoglobulins. Which immunoglobulins are correctly matched to their functions? (Select all that apply)
a. IgA: most responsible for preventing infection in the respiratory tracts, the GI tract, and the genitourinary tract.
b. IgD: provides protection against parasite infestations, especially helminths.
c. IgE: associated with antibody-mediated immediate hypersensitivity reactions.
d. IgG: activates classic complement pathway and enhances neutrophil and
macrophage actions.
e. IgM: first antibody formed by a newly sensitized B-lymphocyte plasma cell.

A

ANS: A,C,D,E
All options are true except IgD acts as a B-cell antigen receptor. IgE provides protection against parasite infestations, especially helminths.

204
Q

The nurse caring for clients assesses their daily laboratory profiles. Which lab results are considered to be in the normal range? (Select all that apply.)

a. Segmented neutrophils: 68%
b. Bands: 19%
c. Monocytes: 12%
d. Lymphocytes: 38%
e. Eosinophils: 2%
f. Basophils: 1%

A

ANS: A,D,E,F
The normal range for segmented neutrophils is 55% to 70%. The normal range for bands is 5%. The normal range for monocytes is 2% to 8%. The normal range for lymphocytes is 20% to 40%. The normal range for eosinophils is 1% to 4%. The normal range for basophils is 0.5% to 1%.

205
Q

What statements about the complement system are correct? (Select all that apply.)

a. Comprised of 20 types of inactive plasma proteins.
b. Act as enzymes when activated to enhance innate immunity.
c. Phagocytize foreign invaders quickly by destroying their membranes.
d. Sticks to the antigen and forms a membrane attack complex.
e. Maintain and prolong inflammation from non-self cells.
f. Is part of the innate immune system.

A

ANS: A,B,D,F
The complement system is made up of 20 different types of inactive plasma proteins that, a membrane attack complex on the antigen surface. This action makes immune cell when activated, act as enzymes to enhance (or complement) cell actions in innate immunity.
They join other proteins to surround antigens and “fix” or stick to the antigen quickly forming
attachment to antigens and phagocytosis more efficient. They are part of innate immunity. They do not phagocytize invaders themselves nor do they maintain and prolong inflammation from allergens.